Vous êtes sur la page 1sur 127

Classes

Quizzes Exams

Solutions

Lecture 1: Mathematical Induction I

Quiz 1

Midterm1

HW 1

Lecture 2: Mathematical Induction II

Quiz 2

Midterm2

HW 2

Lecture 3: Mathematical Induction III

Quiz 3

Final

HW 3

Lecture 4: The Logic of Compound Statements

Quiz 4

HW 4

Lecture 5: Problems About Logical Equivalences

Quiz 5

HW 5

Lecture 6: Valid Argument Forms

Quiz 6

HW 6

Lecture 7: Predicates and Quantifiers Statements

Quiz 7

HW 7

Lecture 8: Set of Problems

Quiz 8

HW 8

Lecture 9: Rational Numbers and Inequalities (Solutions)

Quiz 9

Lecture 10: Problems About Divisibility (Solutions)

Quiz 10

Lecture 11: Set Theory


Lecture 12: Set Theory (Solutions)
Lecture 13: Probability (Problems)
Lecture 14: Counting Elements of Sets I
Lecture 15: Counting Elements of Sets II
Lecture 16: Algebra of Combinations
Lecture 17: Pigeonhole Principle
Lecture 18: Graph Theory

Mathematical Induction
I. Prove by induction the following identities:
n(n + 1)
.
2

1.

1 + 2 + 3 + ... + n =

2.

1 + 3 + 5 + . . . + (2n 1) = n2 .

3.

12 + 22 + 32 + . . . + n2 =

4.

1 2 + 2 3 + 3 4 + . . . + (n 1)n =

5.

1
1
1
1
n1
+
+
+ ... +
=
.
12 23 34
(n 1)n
n

6.

7.
8 .

n(n + 1)(2n + 1)
.
6
n(n 1)(n + 1)
.
3

1
1
1
1
n
+
+
+ ... +
=
.
13 35 57
(2n 1)(2n + 1)
2n + 1



 

1
1
1
1
1
1
1
1
... 1
= .
2
3
4
n
n

1
1
1
1
+
+
+ ... +
= n 1.
n1+ n
1+ 2
2+ 3
3+ 4
sin

9 . sin + sin 2 + sin 3 + . . . + sin n =

n
(n + 1)
sin
2
2
.

sin
2

II. Prove by induction the following inequalities:


1.

n < 2n for any integer n 1.

2.

n2 < n! for any integer n 4.

3.

2n < n! for any integer n 4.

4.

3n < n! for any integer n 7.

5.

nn n! for any integer n 1.

6.

2n+2 2n + 5 for any integer n 1.

7 .

(2n)! < 22n (n!)2 for any integer n 1.

8 .

(n + 1)n < nn+1 for any integer n 3.

1
1
1
1
+
+
+
.
.
.
+
< 2 for any integer n 1.
12 22 32
n2

n
an1 + an2
a1 + a2

10 .

for any positive numbers a1 , a2 and for any integer n 1.


2
2
9 .

III. Prove by induction the following problems:


1.

n3 n is divisible by 3 for any positive integer n.

2.

n5 n is divisible by 5 for any positive integer n.

3.

32n+3 + 40n 27 is divisible by 64 for any positive integer n.

4.

52n+1 2n+2 + 3n+2 22n+1 is divisible by 19 for any positive integer n.

1
5 .
5

!n
1+ 5

!n !
1 5
is integer for any positive integer n.
2

Math175 - Discrete Mathematics - Spring 2005


Quiz #1, March 28, 2005
In the following problems you are required to show all your work and provide the necessary explanations everywhere to get full credit.
1. Use mathematical induction to prove that
4 + 4 5 + 4 52 + 4 53 + . . . + 4 5n = 5n+1 1
for any integer n 0.

2. Use mathematical induction to prove that n2n+1 (n!)2 for any integer n 1.

3. Use mathematical induction to prove that 43n 1 is divisible by 63 for any integer n 0.

1. Use mathematical induction to prove that


4 + 4 5 + 4 52 + 4 53 + . . . + 4 5n = 5n+1 1

()

for any integer n 0.


Proof:
STEP 1: For n=0 () is true, since 4 = 50+1 1.
STEP 2: Suppose () is true for some n = k 0, that is 4+45+452 +453 +. . .+45k = 5k+1 1.
?
STEP 3: Prove that () is true for n = k +1, that is 4+45+452 +453 +. . .+45k +45k+1 =
5k+2 1. We have
ST.2

4 + 4 5 + 4 52 + 4 53 + . . . + 4 5k + 4 5k+1 = 5k+1 1 + 4 5k+1 = 5k+2 1,


which is true, since
?

5k+1 1 + 4 5k+1 = 5k+2 1

?
(1 + 4)5k+1 1 = 5k+2 1

k+1
55
1 = 5k+2 1. 
2. Use mathematical induction to prove that n2n+1 (n!)2 for any integer n 1.
Proof:
STEP 1: For n=1 this is true, since 121+1 = (1!)2 .
STEP 2: Suppose n2n+1 (n!)2 for some n = k 1, that is k 2k+1 (k!)2 .
?

STEP 3: Prove that () is true for n = k + 1, that is (k + 1)2(k+1)+1 ((k + 1)!)2 . We have
ST.2

((k + 1)!)2 = (k!(k + 1))2 = (k!)2 (k + 1)2 k 2k+1 (k + 1)2 (k + 1)2(k+1)+1


which is true, since
?

k 2k+1 (k + 1)2 (k + 1)2(k+1)+1

k 2k+1 (k + 1)2 (k + 1)2k+3

2k+1
k
(k + 1)2k+1 . 
3. Use mathematical induction to prove that 43n 1 is divisible by 63 for any integer n 0.
Proof:
STEP 1: For n=0 this is true, since 430 1 is divisible by 63.
STEP 2: Suppose 43n 1 is true for some n = k 0, that is 63 | 43k 1.
STEP 3: Prove that 63 | 43(k+1) 1. We have
43(k+1) 1 = 43k+3 1 = 43k 64 1 = 43k (63 + 1) 1 = 4| 3k{z 63} +
div. by 63

4| 3k{z 1} . 
St. 2
div. by 63

Math175 - Discrete Mathematics - Spring 2005


Exam #1, April 28, 2005
In the following problems you are required to show all your work and provide the necessary explanations everywhere to get full credit.
I. (10 points) Use mathematical induction to prove that 5n 4n + 1 for any integer n 1.

II. (10 points) Consider various ways of ordering the letters in the word EXCELLENCE:
EEXCLLENCE, LENCEEXCEL, LEXCNCEEEL,
How many distinguishable orderings are there?

and so on.

III. (10 points) Construct a truth table for the statement form ( p q) (q r).

IV. (10 points) Use a truth table to show that the following argument form is valid:
pq
q
r
(p r)

V. (10 points) Use the theorem about logical equivalences to prove that
( p q) (p (p q)) p q.

VI. (10 points) Derive the following set property from those given in the theorem about set
identities and theorem about set properties that involve :
For all sets A and B we have A [(B Ac ) B c ] = .

1.

Exercise set 4.1

2: b0 = 1 + 20 = 2, b1 = 1 + 21 = 3, b2 = 1 + 22 = 5, b3 = 1 + 23 = 9.
i+1
15: One solution is ai = (1)i i+2
for i 0.

21:
34:
36:
55:

P4

1
m=0 2m

Q4
i=1

1
21

1
22

i
i=1 (i+1)!

1
23

1
24

1
1

1
2

1
4

1
8

1
16

31
16

(
(((
n(n1)(nk+1)(
(nk)21
(
(
(
(
(nk)21
(

2.
7:


1 ri .

Pn

n!
(nk)!

1
20

= n(n 1) (n k + 1).

Excercise set 4.2

The formula is true for n = 1:


Certainly, 1 = 1(2 1 1).
If the formula is true for n = k, then it is true for n = k + 1:
Suppose 1 + + (4k 1) = k(2k 1), for some integer k 1. We must
show that 1 + + (4k 1) + (4(k + 1) 1) = (k + 1)(2(k + 1) 1).

But by inductive hypothesis, the left hand side is

k(2k 1) + (4(k + 1) 3) = 2k2 + 3k + 1,

while the right hand side is

(k + 1)(2(k + 1) 1) = 2k2 + 3k + 1.

10: The formula is true for n = 1:


2
Since 13 = 1 = 1(1+1)
.
2
If the formula is true for n = k, then it is true for n = k + 1:
h
i2
for some integer k 1. We compute
Suppose 13 + + k3 = k(k+1)
2
2
k(k + 1)
1 + + k + (k + 1) =
+ (k + 1)3
2

 2
2 k
+ (k + 1)
= (k + 1)
22
3

(k+2)2

z
}|
{
2
k
+
4k
+
4
= (k + 1)2
22

2
(k + 1)(k + 2)
=
,
2

where in the rst equality we have used the inductive hypothesis.

15: The formula is true for n = 2: easy.


If the formula is true for n = k, then it is true for n = k + 1:
k+1 
Y
i=2

1
1 2
i

Y
k 


1
=
1 2
i
i=2




(k + 1)2 1
k+1
=
(k + 1)2
2k

k(k + 2)
(k+1)
(k + 1) + 1

=
=
,


(k+ 1)(k + 1)2
k
2(k + 1)

1
1
(k + 1)2

where the inductive hypothesis has been used in the second equality. 
3.

Excercise set 4.3

9: Clearly, 7 divides 23 1 = 7. We have 23(k+1) 1 = 8 23k 1 = 7 2k + (23k 1),


which is divisible by 7 if we assume 23k 1 is, which we can by inductive
hyphotesis.


10: Again, the base step is trivial (3 divides 3). For the inductive step observe
that the di erence between the term for n = k + 1 and the term for n = k is


(k + 1)3 7(k + 1) + 3 k3 7k + 3
= k3 + 3k2 + 3k + 1 7k 7 + 3 k3 + 7k 3
= 3k2 + 3k 6,

which is divisible by 3. Therefore, if we assume that the term for n = k is


divisible by 3, we conclude that the term for n = k + 1 is divisible by 3 (since
the di erence is).

15: (sketch) The tricky part is to prove that the di erence between two consec-

utive terms,



(k + 1) (k + 1)2 + 5 k k2 + 5 = = 3k2 + 3k + 6,

is divisible by 6. We can work this out again by induction, or note that


3k2 + 3k + 6
k(k + 1)
=
+ 1,
6
2
which is an integer since one of k or k + 1 has to be even.

17: The statement is true for n = 0: 20 = 1 < 2 = (0 + 2)!. Assume it is


true for n = k, for k 0. We claim it is true for n = k + 1. In fact,
2k+1 = 2 2k < 2(k + 2)! by inductive hypothesis; also 2 < k + 3, since k 0;
therefore 2(k + 2)! < (k + 3)(k + 2)! = (k + 3)!.

20: (sketch) The basis step is just a computation. For the inductive step it
suces to see that the di erences between the n = k + 1 inequality and the
n = k inequality are greater (or equal) in the left hand side than in the right

hand side (make sure you understand why this is enough).


(a) (k + 1)3 k3 = 3k2 + 3k + 1, which is bigger than (2(k + 1) + 1) (2k + 1) = 2
(since k 2).

2
2
(b) (k+1)!k! = (k+1)k!k! = kk!, which is bigger than (k+1) k = 2k+1
(since k 4, k k! is certainly bigger than k 3 > 2k + 1).


EXAMPLE 1: Prove that


3 | 4n 1

()

for any integer n 1.


Proof:
STEP 1: For n=1 () is true, since 3 | 41 1.
STEP 2: Suppose () is true for some n = k 1, that is 3 | 4k 1.
STEP 3: Prove that () is true for n = k + 1, that is 4 | 4k+1 1. We have
4k+1 1 = 4 4k 1 = (3 + 1)4k 1 = |3 {z
4k} +
div. by 3

4k 1 . 
| {z }
St. 2
div. by 3

EXAMPLE 2: Prove that


8 | 32n 1

()

for any integer n 1.


Proof:
STEP 1: For n=1 () is true, since 8 | 321 1.
STEP 2: Suppose () is true for some n = k 1, that is 8 | 32k 1.
STEP 3: Prove that () is true for n = k + 1, that is 8 | 32(k+1) 1. We have
32(k+1) 1 = 32k+2 1 = 32k 9 1 = 32k (8 + 1) 1 = 3| 2k{z 8} + 32k 1 . 
| {z }
div. by 8
St. 2
div. by 8
EXAMPLE 3: Prove that
7 | n7 n

()

for any integer n 1.


Proof:
STEP 1: For n=1 () is true, since 7 | 17 1.
STEP 2: Suppose () is true for some n = k 1, that is 7 | k 7 k.
STEP 3: Prove that () is true for n = k + 1, that is 7 | (k + 1)7 (k + 1). We have
(k + 1)7 (k + 1) = k 7 + 7k 6 + 21k 5 + 35k 4 + 35k 3 + 21k 2 + 7k + 1 k 1
=

k 7 k + 7k 6 + 21k 5 + 35k 4 + 35k 3 + 21k 2 + 7k . 


| {z }
|
{z
}
div. by 7
St. 2
div. by 7

EXAMPLE 4: Prove that


1 + 3 + 5 + . . . + (2n 1) = n2

()

for any integer n 1.


Proof:
STEP 1: For n=1 () is true, since 1 = 12 .
STEP 2: Suppose () is true for some n = k 1, that is 1 + 3 + 5 + . . . + (2k 1) = k 2 .
?

STEP 3: Prove that () is true for n = k + 1, that is 1 + 3 + 5 + . . . + (2k 1) + (2k + 1) = (k + 1)2 . We have:
ST.2

1 + 3 + 5 + . . . + (2k 1) + (2k + 1) = k 2 + (2k + 1) = (k + 1)2 . 

EXAMPLE 5: Prove that


1 + 2 + 3 + ... + n =

n(n + 1)
2

()

for any integer n 1.


Proof:
STEP 1: For n=1 () is true, since 1 =

1(1 + 1)
.
2

k(k + 1)
.
2
? (k + 1)(k + 2)
STEP 3: Prove that () is true for n = k + 1, that is 1 + 2 + 3 + . . . + k + (k + 1) =
. We have
2
STEP 2: Suppose () is true for some n = k 1, that is 1 + 2 + 3 + . . . + k =

ST.2

1 + 2 + 3 + . . . + k + (k + 1) =

k(k + 1)
? (k + 1)(k + 2)
+ (k + 1) =
,
2
2

which is true, since


k(k + 1)
? (k + 1)(k + 2)
+ (k + 1) =
2
2

k(k + 1) + 2(k + 1) = (k + 1)(k + 2)

k 2 + k + 2k + 2 = k 2 + 3k + 2. 
EXAMPLE 6: Prove that
12 + 22 + 32 + . . . + n2 =

n(n + 1)(2n + 1)
6

()

for any integer n 1.


Proof:
STEP 1: For n=1 () is true, since 12 =

1(1 + 1)(2 1 + 1)
.
6

k(k + 1)(2k + 1)
.
6
(k + 1)(k + 2)(2k + 3)
STEP 3: Prove that () is true for n = k + 1, that is 12 + 22 + 32 + . . . + k 2 + (k + 1)2 =
.
6
We have
STEP 2: Suppose () is true for some n = k 1, that is 12 + 22 + 32 + . . . + k 2 =

ST.2

12 + 22 + 32 + . . . + k 2 + (k + 1)2 =

k(k + 1)(2k + 1)
? (k + 1)(k + 2)(2k + 3)
+ (k + 1)2 =
,
6
6

which is true, since


k(k + 1)(2k + 1)
? (k + 1)(k + 2)(2k + 3)
+ (k + 1)2 =
6
6

k(k + 1)(2k + 1) + 6(k + 1)2 = (k + 1)(k + 2)(2k + 3)

(k 2 + k)(2k + 1) + 6(k + 1)2 = (k 2 + 3k + 2)(2k + 3)

2k 3 + k 2 + 2k 2 + k + 6k 2 + 12k + 6 = 2k 3 + 3k 2 + 6k 2 + 9k + 4k + 6. 
EXAMPLE 7: Prove that
2n > n
for any integer n 1.
Proof:
STEP 1: For n=1 () is true, since 21 > 1.
STEP 2: Suppose () is true for some n = k 1, that is 2k > k.

()

STEP 3: Prove that () is true for n = k + 1, that is 2k+1 > k + 1. We have


ST.2

2k+1 = 2 2k > 2k k + 1,
which is true, since k 1. 
EXAMPLE 8: Prove that
n! nn

()

for any integer n 1.


Proof:
STEP 1: For n=1 () is true, since 1! = 11 .
STEP 2: Suppose () is true for some n = k 1, that is k! k k .
?

STEP 3: Prove that () is true for n = k + 1, that is (k + 1)! (k + 1)k+1 . We have


ST.2

(k + 1)! = k! (k + 1) k k (k + 1) (k + 1)k+1 ,
which is true, since k k < (k + 1)k . 
I. Prove by induction the following identities:



 

1
1
1
1
1
1.
1
1
1
... 1
= .
2
3
4
n
n
2.

1
1
1
1
n
+
+
+ ... +
=
.
13 35 57
(2n 1)(2n + 1)
2n + 1

3.

1 2 + 2 3 + 3 4 + . . . + (n 1)n =

4 .

n(n 1)(n + 1)
.
3

1
1
1
1
+
+
+ ... +
= n 1.
n1+ n
1+ 2
2+ 3
3+ 4

5 . sin + sin 2 + sin 3 + . . . + sin n =

sin

(n + 1)
n
sin
2
2
.

sin
2

II. Prove by induction the following inequalities:


1.

2n < n! for any integer n 4.

2.

3n < n! for any integer n 7.

3.

3n 2n + 1 for any integer n 1.

4.

2n+2 2n + 5 for any integer n 1.

5 .

(2n)! < 22n (n!)2 for any integer n 1.

6 .

(n + 1)n < nn+1 for any integer n 3.

7 .

an1 + an2

8 .

1
1
1
1
+ 2 + 2 + . . . + 2 < 2 for any integer n 1.
12
2
3
n

a1 + a2
2

n
for any positive numbers a1 , a2 and for any integer n 1.

III. Prove by induction the following problems:


1.

n3 n is divisible by 3 for any nonnegative integer n.

2.

n5 n is divisible by 5 for any nonnegative integer n.

3.

n3 7n + 3 is divisible by 3 for any nonnegative integer n.

4.

4n 1 is divisible by 3 for any nonnegative integer n.

5.

32n 1 is divisible by 8 for any positive integer n.

6 .

32n+3 + 40n 27 is divisible by 64 for any nonnegative integer n.

7 .

52n+1 2n+2 + 3n+2 22n+1 is divisible by 19 for any nonnegative integer n.

8 .

!n
1+ 5

!n !
1 5
is integer for any nonnegative integer n.
2

Math175 - Discrete Mathematics - Spring 2005


Quiz #2, March 31, 2005
In the following problems you are required to show all your work and provide the necessary explanations everywhere to get full credit.
1. Use mathematical induction to prove that
1
1
1
1
+ 2 + ... + 2 < 1
2
2
3
n
n
for any integer n 2.

2. Use mathematical induction to prove that 7 4n3 < n! for any integer n 5.

3. Use mathematical induction to prove that 72n 1 is divisible by 48 for any integer n 0.

1. Use mathematical induction to prove that


1
1
1
1
()
+ 2 + ... + 2 < 1
2
2
3
n
n
for any integer n 2.
Proof:
1
1
STEP 1: For n=2 () is true, since 2 < 1 .
2
2
1
1
1
1
STEP 2: Suppose () is true for some n = k 2, that is 2 + 2 + . . . + 2 < 1 .
2
3
k
k
1
1
1
1
1
STEP 3: Prove that () is true for n = k + 1, that is 2 + 2 + . . . + 2 +
.
< 1
2
2
3
k
(k + 1)
k+1
We have
ST.2
?
1
1
1
1
1
1
1
,
+ 2 + ... + 2 +
< 1 +
<1
2
2
2
2
3
k
(k + 1)
k (k + 1)
k+1
which is true, since
?
1
1
1
1 +
<1
2
k (k + 1)
k+1

?
k+1
(1 + 4)5
1 = 5k+2 1

5 5k+1 1 = 5k+2 1. 
2. Use mathematical induction to prove that n2n+1 (n!)2 for any integer n 1.
Proof:
STEP 1: For n=1 this is true, since 121+1 = (1!)2 .
STEP 2: Suppose n2n+1 (n!)2 for some n = k 1, that is k 2k+1 (k!)2 .
?

STEP 3: Prove that () is true for n = k + 1, that is (k + 1)2(k+1)+1 ((k + 1)!)2 . We have
ST.2

((k + 1)!)2 = (k!(k + 1))2 = (k!)2 (k + 1)2 k 2k+1 (k + 1)2 (k + 1)2(k+1)+1


which is true, since
?

k 2k+1 (k + 1)2 (k + 1)2(k+1)+1

k 2k+1 (k + 1)2 (k + 1)2k+3

2k+1
k
(k + 1)2k+1 . 
3. Use mathematical induction to prove that 43n 1 is divisible by 63 for any integer n 0.
Proof:
STEP 1: For n=0 this is true, since 430 1 is divisible by 63.
STEP 2: Suppose 43n 1 is true for some n = k 0, that is 63 | 43k 1.
STEP 3: Prove that 63 | 43(k+1) 1. We have
43(k+1) 1 = 43k+3 1 = 43k 64 1 = 43k (63 + 1) 1 = 4| 3k{z 63} +
div. by 63

4| 3k{z 1} . 
St. 2
div. by 63

Math175 - Discrete Mathematics - Spring 2005


Exam #2, May 20, 2005
In the following problems you are required to show all your work and provide the necessary explanations everywhere to get full credit.
I. (30 points)
     
7
10
200
(a) Evaluate
,
,
.
4
7
199

 
n
n(n 1)
.
(b) Show that
=
2
2

         
 
10
10
10
10
10
10
(c) Show that

+
... +
= 0.
0
1
2
3
4
10

II. (20 points)


Statement

True

Not true

The set of all positive integers is uncountable


The set of all real numbers is countable
The set of all rational numbers is countable
The set of all positive rational numbers is countable
The set of all real numbers from (0, 1) is uncountable
The set of all real numbers has the same cardinality as the set of all rationals
The set of all real numbers from (0, 1) has the same cardinality
as the set of all real numbers from (0, 2)
The set of all rational numbers from (0, 1) has the same cardinality
as the set of all rational numbers from (0, 2)
The set of all integer numbers from (0, 10) has the same cardinality
as the set of all integer numbers from (0, 20)
The set of all real numbers from (0, 10) has the same cardinality
as the set of all rational numbers from (0, 20)

III. (10 points) Let A = {10, 13, 16, 19, 22, 25, . . . , 73}. Prove that if 12 integers are selected
from A, then at least one pair of integers has a sum of 83.

IV. (10 points) Find a sequence that satisfies the recurrence relation ak = ak1 + 2ak2 for
all integers k 2 and that also satisfies the initial conditions a0 = 2 and a1 = 7.

V. (10 points)
Statement
3x8 5x3 + 1 is O(x8 )
3x8 5x3 + 1 is O(x9 )
3x8 5x3 + 1 is (x3 )
3x3 5x + 1 is (x8 )
3x3 5x + 1 is (x4 )

True

Not true

VI. (10 points) Let A = {7, 10, 15, 40} and B = {4, 8, 22, 27, 28} and let R be the following
relation:
For all (x, y) A B, x R y x > y.
State explicitly which ordered pairs are in R and R1 .

VII. (30 points)


Draw a graph with the specified properties or show that no such graph exists.
(a) Graph with five vertices of degrees 1, 2, 2, 2, and 3.
(b) Graph with five vertices of degrees 1, 1, 2, 3, and 4.
(c) Simple graph with four vertices of degrees 1, 2, 3, and 4.

1. Exercise set 1.1

7: m c.
13:

Here is the truth table:

p q

p q (p q) (p q) (p q)

15:

Here is the truth table:

p r qr

p (q r)

p q

22:

The two statements

are

logically equivalent:

qr pq pr

p (q r) (p q) (p r)

p q

24:

The two statements are

pq pr

not

logically equivalent:

(p q) (p r) (p q) r

p q

*
*
*

45:
(8)

(p q) ( p q) ( p q) ( p q)
(3)

(5)

(4)

p (q q) p t p.

2. Excercise set 1.2

13a: Here is the truth table:


p q p p q pq
T

21:

A conditional statement is

p q

p q

not

logically equivalent to its inverse:

conditional inverse
pq
p q

23:

The converse and inverse of a conditional statement

p q

p q

are

logically equivalent:

converse inverse
q p p q

30:
p q r q ( p q) (r q) = ?;
(b) ? (p q) ( r q) ((p q) ( r q)).
(a)

35:
39:

If this triangle has two

45

angles, then it is a right triangle.

If this computer program is correct, then it does not produce error messages.

EXAMPLE 1: Prove that


3 | n3 n

()

for any integer n 0.


Proof:
STEP 1: For n=0 () is true, since 3 | 03 0.
STEP 2: Suppose () is true for some n = k 0, that is 3 | k 3 k.
STEP 3: Prove that () is true for n = k + 1, that is 3 | (k + 1)3 (k + 1). We have
(k + 1)3 (k + 1) = k 3 + 3k 2 + 3k + 1 k 1 =

k 3 k + 3k 2 + 3k . 
| {z }
| {z }
div. by 3
St. 2
div. by 3

EXAMPLE 2: Prove that


5 | n5 n

()

for any integer n 0.


Proof:
STEP 1: For n=0 () is true, since 5 | 05 0.
STEP 2: Suppose () is true for some n = k 0, that is 5 | k 5 k.
STEP 3: Prove that () is true for n = k + 1, that is 5 | (k + 1)5 (k + 1). We have
(k + 1)5 (k + 1) = k 5 + 5k 4 + 10k 3 + 10k 2 + 5k + 1 k 1 =

k 5 k + 5k 4 + 10k 3 + 10k 2 + 5k . 
|
| {z }
{z
}
div. by 5
St. 2
div. by 5

EXAMPLE 3: Prove that


3 | n3 7n + 3

()

for any integer n 0.


Proof:
STEP 1: For n=0 () is true, since 3 | 03 7 0 + 3.
STEP 2: Suppose () is true for some n = k 0, that is 3 | k 3 7k + 3.
STEP 3: Prove that () is true for n = k + 1, that is 3 | (k + 1)3 7(k + 1) + 3. We have
(k + 1)3 7(k + 1) + 3 = k 3 + 3k 2 + 3k + 1 7k 7 + 3 = k 3 7k + 3+ 3k 2 + 3k 6 . 
|
{z
} |
{z
}
div.
by
3
St. 2
div. by 3
EXAMPLE 4: Prove that


1
1
2



 

1
1
1
1
1
1
... 1
=
3
4
n
n

()

for any integer n 2.


Proof:

STEP 1: For n=2 () is true, since

1
1
2


=

1
.
2




 

1
1
1
1
1
STEP 2: Suppose () is true for some n = k 2, that is 1
1
1
... 1
= .
2
3
4
k
k







1
1
1
1
1
?
STEP 3: Prove that () is true for n = k + 1, that is 1
1
1
... 1
1
=
2
3
4
k
k+1
1
. We have:
k+1









1
1
1
1
1
1
1
ST.2 1
?
1
1
1
... 1
1
=
1
=
,
2
3
4
k
k+1
k
k+1
k+1

Approach I: which is true, since



1
1
?
1
=
k+1
k+1


1 k+1
1
1
?

=
k k+1 k+1
k+1


1 k+11 ? 1
=
k
k+1
k+1



1
k
1
?
=
k k+1
k+1

k
1
=
.
k(k + 1)
k+1
1
k

Approach II: which is true, since



1
1
?
1
=
k+1
k+1


k
1
k
?
1
=
k
k+1
k+1

1 ? k
1
=
k+1
k+1

k + 1 ? k(k + 1)
=
(k + 1)
k+1
k+1

k + 1 1 = k. 
1
k

EXAMPLE 5: Prove that


1
1
1
1
n
+
+
+ ... +
=
13 35 57
(2n 1)(2n + 1)
2n + 1

()

for any integer n 1.


Proof:
STEP 1: For n=1 () is true, since

1
1
=
.
13
21+1

1
1
1
1
k
+
+
+...+
=
.
13 35 57
(2k 1)(2k + 1)
2k + 1
1
1
1
1
STEP 3: Prove that () is true for n = k + 1, that is
+
+
+ ... +
+
13
35
57
(2k 1)(2k + 1)
1
k+1
?
=
. We have
(2(k + 1) 1)(2(k + 1) + 1)
2(k + 1) + 1

STEP 2: Suppose () is true for some n = k 1, that is

1
1
1
1
1
+
+
+ ... +
+
13 35 57
(2k 1)(2k + 1) (2(k + 1) 1)(2(k + 1) + 1)
1
1
1
1
1
=
+
+
+ ... +
+
13 35 57
(2k 1)(2k + 1) (2k + 2 1)(2k + 2 + 1)
1
1
1
1
1
=
+
+
+ ... +
+
13 35 57
(2k 1)(2k + 1) (2k + 1)(2k + 3)
k
1
k+1
ST.2
?
=
+
=
,
2k + 1 (2k + 1)(2k + 3)
2(k + 1) + 1

which is true, since


k
1
k+1
?
+
=
2k + 1 (2k + 1)(2k + 3)
2(k + 1) + 1

k
1
? k+1
+
=
2k + 1 (2k + 1)(2k + 3)
2k + 3

k(2k + 3)
1
? k+1
+
=
(2k + 1)(2k + 3) (2k + 1)(2k + 3)
2k + 3

k(2k + 3) + 1 ? k + 1
=
(2k + 1)(2k + 3)
2k + 3

k(2k + 3) + 1 ?
=k+1
2k + 1

k(2k + 3) + 1 = (k + 1)(2k + 1)

2k 2 + 3k + 1 = 2k 2 + k + 2k + 1. 
EXAMPLE 6: Prove that
1 2 + 2 3 + 3 4 + . . . + (n 1)n =

n(n 1)(n + 1)
3

()

for any integer n 2.


Proof:
STEP 1: For n=2 () is true, since 1 2 =

2(2 1)(2 + 1)
.
3

k(k 1)(k + 1)
.
3
(k + 1)k(k + 2)
STEP 3: Prove that () is true for n = k +1, that is 12+23+34+. . .+(k 1)k +k(k +1) =
.
3
We have
STEP 2: Suppose () is true for some n = k 2, that is 1 2 + 2 3 + 3 4 + . . . + (k 1)k =

ST.2

1 2 + 2 3 + 3 4 + . . . + (k 1)k + k(k + 1) =

k(k 1)(k + 1)
? (k + 1)k(k + 2)
+ k(k + 1) =
,
3
3

which is true, since


k(k 1)(k + 1)
? (k + 1)k(k + 2)
+ k(k + 1) =
3
3

k(k 1)(k + 1) + 3k(k + 1) = (k + 1)k(k + 2)

(k 2 k)(k + 1) + 3k 2 + 3k = (k 2 + k)(k + 2)

k 3 + k 2 k 2 k + 3k 2 + 3k = k 3 + 2k 2 + k 2 + 2k. 
EXAMPLE 7: Prove that
2n < n!

()

for any integer n 4.


Proof:
STEP 1: For n=4 () is true, since 24 < 4! (16 < 24).
STEP 2: Suppose () is true for some n = k 4, that is 2k < k!.
?

STEP 3: Prove that () is true for n = k + 1, that is 2k+1 < (k + 1)!. We have
ST.2

2k+1 = 2 2k < 2k! < (k + 1)!,

which is true, since


?

2k! < (k + 1)!

2k! < k!(k + 1)

2<k+1

1 < k. 
EXAMPLE 8: Prove that
3n < n!

()

for any integer n 7.


Proof:
STEP 1: For n=7 () is true, since 37 < 7! (2187 < 5040).
STEP 2: Suppose () is true for some n = k 7, that is 3k < k!.
?

STEP 3: Prove that () is true for n = k + 1, that is 3k+1 < (k + 1)!. We have
ST.2

3k+1 = 3 3k < 3k! < (k + 1)!,


which is true, since
?

3k! < (k + 1)!

3k! < k!(k + 1)

3<k+1

2 < k. 
EXAMPLE 9: Prove that
3n 2n + 1

()

for any integer n 1.


Proof:
STEP 1: For n=1 () is true, since 31 = 2 1 + 1.
STEP 2: Suppose () is true for some n = k 1, that is 3k 2k + 1.
STEP 3: Prove that () is true for n = k + 1, that is 3k+1 2(k + 1) + 1. We have
ST.2

3k+1 = 3 3k 3(2k + 1) 2(k + 1) + 1,


which is true, since
?

3(2k + 1) 2(k + 1) + 1

6k + 3 2k + 2 + 1

4k 0. 
EXAMPLE 10: Prove that
2n+2 2n + 5
for any integer n 1.
Proof:

()

STEP 1: For n=1 () is true, since 21+2 2 1 + 5.


STEP 2: Suppose () is true for some n = k 1, that is 2k+2 2k + 5.
STEP 3: Prove that () is true for n = k + 1, that is 2k+3 2(k + 1) + 5. We have
ST.2

2k+3 = 2 2k+2 3(2k + 5) 2(k + 1) + 5,


which is true, since
?

3(2k + 5) 2(k + 1) + 5

6k + 15 2k + 2 + 5

4k + 8 0. 

Math175 - Discrete Mathematics - Spring 2005


Quiz #3, April 4, 2005
In the following problems you are required to show all your work and provide the necessary explanations everywhere to get full credit.
Use the theorem about logical equivalences to prove that
(p q) ( p (p q)) t.

Math175 - Discrete Mathematics - Spring 2005


Final Exam, May 31, 2005
In the following problems you are required to show all your work and provide the necessary explanations everywhere to get full credit.
I. (10 points)
Use mathematical induction to prove that

n
X
m=1

m 2m = 2 + (n 1)2n+1 for any integer n 1.

II. (10 points)


Construct a truth table for the statement form ( p q) (q r).

III. (10 points)


Use the theorem about logical equivalences to prove that
( p q) (p (p q)) p q.

IV. (10 points)


Use a truth table to show that the following argument form is valid:
pq
q
r
(p r)

V. (15 points)
Let U = {1, 2, 3, . . . , 10} and let A = {1, 2, 3, 4, 5}, B = {1, 2, 4, 8}, C = {1, 2, 3, 5, 7}, and
D = {2, 4, 6, 8}. Determine each of the following:
(a) (A B) C

(b) C c Dc

(c) A (B C)

(d) (B C) D

(e) (A B) (C D)

(f) (A B) (C D)c

(g) (A B)c (C D)

VI. (10 points)


Derive the following set property from those given in the Theorem About Set Identities:
For all sets A and B we have ([(A B) C]c B c )c = B C c .

VII. (20 points)


Let T = {4, 6, 8, 10, 12, 14, 16, 18, 20}.
(a) Suppose 5 integers are chosen from T. Must there be two integers whose sum is 24?

(b) Suppose 6 integers are chosen from T. Must there be two integers whose sum is 24?

VIII. (15 points)


Give the contrapositive, converse and inverse of the following quantified statement:
integers a, b, and c, if a b is even and b c is even, then a c is even.

IX. (10 points)


A team of 7 persons is to be selected from 10 people, two of whom refuse to work together. In
how many ways can the team be selected?

X. (10 points)
Use the definition of O-notation to show that 10x3 + 7x + 3 is O(x3 ).

XI. (20 points)


Draw a graph with the specified properties or show that no such graph exists.
(a) Graph with five vertices of degrees 1, 2, 2, 2, and 3.
(b) Graph with five vertices of degrees 2, 2, 2, 2, and 3.
(c) Graph with four vertices of degrees 1, 2, 2, and 3.
(d) Simple graph with four vertices of degrees 1, 2, 2, and 3.

1.

Exercise set 1.3

11: The argument is valid, since whenever the premises are both true, the con-

clusion is true:
p q

T
T
F
F

premises

conclusion

p q q

T
F
T
F

T
F
T
T

F
T
F
T

F
F
T
T

12:

(b) This argument is not valid, because there is a case where both premises
are true, but the conclusion is false:
p q

T
T
F
F

premises

conclusion

p q p

T
F
T
F

T
F
T
T

F
F
T
T

F
T
F
T

19: The truth table shows that whenever the premises are both true, the con-

clusion is true, so the argument is valid:


p q

T
T
T
T
F
F
F
F

T
F
T
F
T
F
T
F

T
T
F
F
T
T
F
F

premises

conclusion

pq qr

pr

T
T
F
F
T
T
T
T

T
F
T
T
T
F
T
T

T
F
T
F
T
T
T
T

*
*
*

25: The argument is


m h,
h e
m e

where m stands for \I go to the movies", h stands for \I will nish my homework", and e stands for \I will do well on the exam tomorrow".
It is valid by the rule of hypothetical syllogism.
2.

Excercise set 2.1

5: For a = 1 Z, we have (a 1)/a = 2, which

is

an integer.

11:

(b) real number x, x is positive, negative, or zero.


12:

(b) a real number x such that x is rational.


25: Not correct. Correct negation:

There is an irrational number and a rational number whose product is rational.

30: a computer program P, such that P is correct and P doesn't compile without
error messages).

3.

Excercise set 2.2

4: There is a book that everybody has read.


17: (a) integer x, ( a prime number p: x p 2x).
(b) an integer x: ( prime number p, p < x or 2x p).
19: (a) x R, y R such that x > y.

(b) both statements are true. For the original statement, take x to be a nonnegative number, which is certainly bigger than any negative number. For the
statement in (a), for every x R, if x is negative one can take y = x 1, which

is certainly smaller than x, whereas if x is nonnegative, any y R will be


smaller.
25: statement: integer x, if x2 is even, then x is even.
contrapositive: integer x, if x is odd, then x2 is odd.
converse: integer x, if x is even, then x2 is even.
inverse: integer x, if x2 is odd, then x is odd.
32: If a number is divisible by 6, then it is divisible by 3.

4.
4:


1 4

52

Excercise set 2.3

= 5( 2 4) .
1

12: The argument is invalid. It exhibits the inverse error.


15: The argument is invalid. It exhibits the converse error.

DEFINITION: If p and q are statement variables, the conditional of q by p is If p then q


or p implies q and is denoted p q. It is false when p is true and q is false; otherwise it is
true. We call p the hypothesis of the conditional and q the conclusion.
p
T
T
F
F

pq
T
F
T
T

q
T
F
T
F

EXAMPLE: Construct a truth table for the following conditional form:


p q p

Basic Logical Equivalences Involving


1. p q q p
2. q p p q
3. p q r (p r) (q r)
4. p q p q
5. (p q) p q

DEFINITION: Given statement variables p and q, the biconditional of p and q is p if, and
only if, q and is denoted p q. It is true if both p and q have the same truth values and is
false if p and q have opposite truth values.
p
T
T
F
F

pq
T
F
F
T

q
T
F
T
F

DEFINITION: An argument form is a sequence of statements. All statements but the final
one are called premises (or assumptions or hypotheses). The final statement is called the
conclusion.
EXAMPLE:
If the last digit of this number is a two, then this number is divisible by two.
The last digit of this number is a two.
Therefore this number is divisible by two.
1

EXAMPLE:
If the last digit of this number is even, then this number is divisible by two.
If this number is divisible by two, then the last digit of this number is even.
Therefore the last digit of this number is even or this number is divisible by two.
DEFINITION: To say that an argument form is valid means that no matter what particular
statements are substituted for the statement variables in the premises, if the resulting premises
are all true, then the conclusion is also true.

Valid Argument Forms


pq
p
q
Modus tollels
pq
q
p
Disjunctive
p
addition I
pq
Disjunctive
q
addition II
pq
Conjunctive
pq
simplification I
p
Conjunctive
pq
simplification II
q
Conjunctive
p
addition
q
pq
Disjunctive
pq
syllogism I
q
p
Disjunctive
pq
syllogism II
p
q
Hypothetical
pq
syllogism
qr
pr
Dilemma
pq
pr
qr
r
Rule of
pc
contradiction
p
Modus ponens

if this number is divisible by 10, then this number is divisible by 5


this number is divisible by 10
therefore this number is divisible by 5
if this number is divisible by 10, then this number is divisible by 5
this number is NOT divisible by 5
therefore this number is NOT divisible by 10
this number is divisible by 10
therefore this number is divisible by 10 or by 7 (by ...)
this number is divisible by 7
therefore this number is divisible by 10 (by ...) or by 7
this number is divisible by 2 and by 3
therefore this number is divisible by 2
this number is divisible by 2 and by 3
therefore this number is divisible by 3
this number is divisible by 9
this number is divisible by 12
therefore this number is divisible by 9 and by 12
this number is divisible by 3 or by 7
this number is NOT divisible by 7
therefore this number is divisible by 3
this number is divisible by 3 or by 7
this number is NOT divisible by 3
therefore this number is divisible by 7
if this number is divisible by 12, then this number is divisible by 6
if this number is divisible by 6, then this number is divisible by 3
therefore if this number is divisible by 12, then this number is divisible by 3
this number is divisible by 8 or by 12
if this number is divisible by 8, then this number is divisible by 4
if this number is divisible by 12, then this number is divisible by 4
therefore this number is divisible by 4
if this number is NOT divisible by 10, we obtain a contradiction
therefore this number is divisible by 10

Math175 - Discrete Mathematics - Spring 2005


Quiz #4, April 11, 2005
In the following problems you are required to show all your work and provide the necessary explanations everywhere to get full credit.
Use a truth table to show that the following argument form is valid:
pq
pr
qr
r

1.

Exercise set 3.1

2b: Yes: 6r + 4s2 + 3 = 2(3r + 2s2 + 1) + 1, where 3r + 2s2 + 1 is an integer (since


r and s are integers). Therefore, it is odd, by de nition.

4: Yes: for a = b = 0, we have 0 + 0 = 0 = 0 + 0.


9: The values of n2 n+11 for 1 n 10 are (in order): 11, 13, 17, 23, 31, 41, 53, 67, 83, 101,
which are all prime numbers.

12: Let m and n be two odd numbers. By de nition, m = 2k + 1 and n = 2l + 1,
for some integers k and l. Adding the two equations yields: m + n = 2k + 1 +
2l + 1 = 2(k + l + 1), which is even, by de nition.

16: For instance, take a = 5, b = 2. Then a < b, but a2 = 25 4 = b2 .
35: For instance, take n = 11. Then n2 n + 11 = 112 , which is not prime.

2.

Exercise set 3.2

2: 39602/10000 = 19801/5000
15: Let m =

a
b

and n =

c
d

mn=

be any two rational numbers. The di erence is


a c
ad bc
ad bc
=

=
,
b d
bd bd
bd

which is a rational number.

Verify the following logical equivalences:


1.

(p q) p p

2.

(p q) (p q) p

3.

( p q) (p q) p

4.

p ( q p) p

5.

(p q) ( p q) q

6.

(p q) ( p q) c

7.

(p q) [ p (p q)] t

8.

[( p q) (p r)] ( p q) (p r)

9.

(r p) [( r (p q)) (r q)] p q

10. [( p q) ( p q)] (p q) p.

Math175 - Discrete Mathematics - Spring 2005


Quiz #5, April 18, 2005
In the following problems you are required to show all your work and provide the necessary explanations everywhere to get full credit.
1. Give the contrapositive, converse and inverse of the following quantified statement:
integers a, b, and c, if a b is even and b c is even, then a c is even.

2. Negate the following multiply quantified statement P :


x R, y R | 2x + 3y = 1.
Determine, which statement is false: P or P.

1.

Exercise set 3.2

28: Suppose c is a root of a (nonzero) polynomial with rational coecients:


P(c) =

a1
a0
an n
c + + c +
= 0,
bn
b1
b0

where ai , bi Z, bi 6= 0, and an 6= 0. We can multiply this equation by


bn b1 b0 6= 0, to get:
P 0 (c) = dn cn + + d1 c + d0 = 0,

where di = bn b1 b0 abii is clearly an integer for all i, and dn 6= 0. Thus, c is


a root of the (nonzero) polynomial P 0 (X), whose coecients are integers. 
29: This is \arguing from examples": it is proving the statement for a particular

case.

32: This is \begging the question": it is assuming that r + s is rational, and


using it to prove that r + s is rational.

2.

Exercise set 3.3

4: Yes: 2m(2m + 2) = 2m 2(m + 1) = 4m(m + 1), which is divisible by 4 since


m(m + 1) is an integer.
12: Yes: n2 1 = (4k + 3)2 1 = 16k2 + 24k + 9 1 = 8(2k2 + 3k + 1).
15: Let a, b and c be integers, and assume a | b and a | c. By de nition, there
are integers r and s such that b = ar, c = as. Therefore, bc = aras =
a (r s), with r s an integer. We conclude that a | b c.
24: False. For instance, let a = 6, b = 2, and c = 3. Then a | bc, but neither
a | b nor a | c. (The statement is true if we ask for a to be a prime number.)

3.
2: 56 = 5 11 + 1.

Exercise set 3.4

18: Let m and m + 1 be the two consecutive integers. We have two cases:
Case 1: m even. Then m = 2k for some integer k, and m(m+1) = 2k(2k+1)

is even, by de nition.
Case 2: m odd. Then m = 2k + 1 for some integer k, and m(m + 1) =
(2k + 1)(2k + 2) = 2(2k + 1)(k + 1), which is also even.
Therefore, m(m + 1) is even.


20: Apply the Quotient-Remainder Theorem with d = 3. This implies that


there exist integers q and r such that n = 3q + r, and 0 r < 3. But the
only nonnegative integers less than 3 are 0, 1, and 2. Hence, either n = 3q,
n = 3q + 1, or n = 3q + 2.

28: By the Quotient-Remainder Theorem, there exist integers 1 and r sucn that
n = 4q + 4, with 0 r < 4. Now, n(n2 1)(n + 2) = (n 1)n(n + 1)(n + 2),
and its easy to see that for each possible value of r, exactly one of the factors
is divisible by 4.1 Hence, their product is always divisible by 4.


4.

Exercise set 3.6

4: Negation: \There is a positive rational number x, which is the least positive


rational number, i.e. for any positive rational number y, x y". Suppose such
an x exists. Then, for all positive rational numbers y, x y. In particular, for
y = x/2, we have x x/2, which is a contradiction (since x is positive).


21: False. For instance, 2 is irrational, and 2 2 = 2, which is rational.


22: Suppose that c = a + br were rational. Say a = aa21 , b =
a1 , a2 , b1 , b2 , c1 , c2 Z. Since b 6= 0, we can write
ca
r=
=
b

c1
c2

b1
b2

a1
a2

c1 a2 c2 a1
c2 a2
b1
b2

b1
b2

, and c =

c1
c2

, with

b2 (c1 a2 c2 a1 )
,
b1 c2 a2

which is rational, by de nition (note that c2 6= 0 and a2 6= 0 for they are


denominators of rational numbers, while b1 6= 0 since b 6= 0.)

1 If
by

4;

r = 0, then n is divisible by 4; if r = 1, then n 1 is divisible by 4;


r = 3, then n + 1 is divisible by 4 (prove it!).

and if

if

r = 2, then n + 2 is divisible

Valid Argument Forms


pq
if n is divisible by 10, then n is divisible by 5
p
n is divisible by 10
q
therefore n is divisible by 5
Modus tollels
pq
if n is divisible by 10, then n is divisible by 5
q
n is NOT divisible by 5
p
therefore n is NOT divisible by 10
Disjunctive
p
n is divisible by 10
addition I
pq
therefore n is divisible by 10 or by 7 (by ...)
Disjunctive
q
n is divisible by 7
addition II
pq
therefore n is divisible by 10 (by ...) or by 7
Conjunctive
pq
n is divisible by 2 and by 3
simplification I
p
therefore n is divisible by 2
Conjunctive
pq
n is divisible by 2 and by 3
simplification II q
therefore n is divisible by 3
Conjunctive
p
n is divisible by 9
addition
q
n is divisible by 12
pq
therefore n is divisible by 9 and by 12
Disjunctive
pq
n is divisible by 3 or by 7
syllogism I
q
n is NOT divisible by 7
p
therefore n is divisible by 3
Disjunctive
pq
n is divisible by 3 or by 7
syllogism II
p
n is NOT divisible by 3
q
therefore n is divisible by 7
Hypothetical
pq
if n is divisible by 12, then n is divisible by 6
syllogism
qr
if n is divisible by 6, then n is divisible by 3
p r therefore if n is divisible by 12, then n is divisible by 3
Dilemma
pq
n is divisible by 8 or by 12
pr
if n is divisible by 8, then n is divisible by 4
qr
if n is divisible by 12, then n is divisible by 4
r
therefore n is divisible by 4
Rule of
p c if n is NOT divisible by 10, we obtain a contradiction
contradiction
p
therefore n is divisible by 10
Modus ponens

Math175 - Discrete Mathematics - Spring 2005


Quiz #6, April 27, 2005
In the following problems you are required to show all your work and provide the necessary explanations everywhere to get full credit.
1. The following is a proof that for any sets A and B,
B A = B Ac .
Fill in the blanks.
and
are any sets.
Proof: Suppose
Step 1. We first prove that B A
.
Suppose x
. From this by definition of set difference it follows that x
and
. But then by definition of complement x
and x
, and so by definition
x 6
of
we have x B Ac .
.
Step 2. We now prove that
. From this by definition of
it follows that x
and
Suppose x
x
. But then by definition of complement x
and x 6
, and so by definition
.
of set difference we have

2. Derive the following set property from those given in the Theorem About Set Identities and
Theorem About Set Properties That Involve :
For all sets A and B we have (B A) (A B) = A.


 


3: {0, 1, 2} = {x Z : 1 < x < 3}



6: 
 
$ 
( 

9: 


3 {1, 2, 3}

{3} {1, {2} , {3}}

{1} {1, 2}

{1} {1}

{x R
{x R
{x R
{x R

 

)

$ 

14: 

)

:
:
:
:


!
%


1 * {1}

1 {1}

1 6 {{1} , 2}


2 x < 3}

x < 2 1 < x}

x < 2 3 x}

x 1 1 < x}


 
"#
%


"#
&'

{x R
{x R
{x R
{x R


{2} 6 {1, 2}

{2} * {1, {2} , {3}}

{1} {1, {2}}

:
:
:
:


1 < x 1}

x 1 3 x}

x 1 1 < x}

x < 2 3 x}


A (B C) = (A B) (A C) = {a, b, c} =
6
(A B) C = {b, c}

A (B C) = (A B) (A C) = {b, c}
6
=
(A B) C = {b, c, e}

(A B) C = {a}
6
=
A (B C) = {a, b, c}

15:

C




A (B C)

C


BC

C
 

AB

C
!

(A B)c

Ac

C
"#

Ac Bc

18: 

A (B C) =

{(1, (u, m)), (1, (u, n)), (1, (v, m)), (1, (v, n)),
(2, (u, m)), (2, (u, n)), (2, (v, m)), (2, (v, n)),
(3, (u, m)), (3, (u, n)), (3, (v, m)), (3, (v, n))}
(A B) C) = {((1, u), m), ((1, u), n), ((1, v), m), ((1, v), n),
((2, u), m), ((2, u), n), ((2, v), m), ((2, v), n),
((3, u), m), ((3, u), n), ((3, v), m), ((3, v), n)}
ABC=
{(1, u, m), (1, u, n), (1, v, m), (1, v, n),
(2, u, m), (2, u, n), (2, v, m), (2, v, n),
(3, u, m), (3, u, n), (3, v, m), (3, v, n)}



)

*+
 -,


2E
HG'IKJMLKN?OPQOSRT)UVP'RW?P
4: 0
B 6B A@:CD> >FA@
A B
. /21!1436587:9;<;=?> A A@
 2f &gES&
2E
\G'IKJKL]N^O_P`OaRT!UbP'RW?P
"
@
=!@h=
A B B X Y[Z
x AB
x B X YdceC
&
8j 
Ek%

& _
8j 
9
@ =!@i x A =)B x B @ > x A i
@M> @
A B i x B @ > x B i
ES%
_lg l

& &mEk%

GmWnOoUpWnOqP#TsrJtOaRuT)Upv
@
6B C x B 7:= @
B > i x B
YaX


11: wBk9

)

.0/21!1436x

x (A B) (C B) x (A B) x (C B)
(x A x 6 B) (x C x 6 B)
(x A x C) x 6 B
x A C x 6 B
x (A C) B.

l )z{ $ p
 "

= B A = {1} , B = {} , C = {}
)
12: y >

31: |~}

2"

B @

_
>

=w

=)B

_b+*+*?
(10)

(2)

(10)

(2)

(A B) C = (A Bc ) Cc = A (Bc Cc )
(A C) B = (A Cc ) Bc = A (Cc Bc ),
A@

 ES%
E~

=6B

qn

9<

lu

C (1)

34: |~}

_"

B @

_

E
>

=w

=)B

b+*+*?
(10)

(2)

(A B) C = (A Bc ) Cc = A (Bc Cc )
(10)

(7)

A (B C) = A (B C)c = A (Bc Cc ).
36: |~}

2"

B @

_
>

=w

=)B

_b+*+*?
(10)

(9)

(5)

(Bc (Bc A))c = (Bc (Bc Ac ))c = (Bc )c = B.

DEFINITION:
A predicate is a sentence that contains a
finite number of variables and becomes a
statement when specific values are substituted for the variables.

DEFINITION:
A predicate is a sentence that contains a
finite number of variables and becomes a
statement when specific values are substituted for the variables.
Example: n divides 8

DEFINITION:
A predicate is a sentence that contains a
finite number of variables and becomes a
statement when specific values are substituted for the variables.
Example: n divides 8

DEFINITION:
The domain D of a predicate variable is
the set of all values that may be substituted in place of the variable.

DEFINITION:
A predicate is a sentence that contains a
finite number of variables and becomes a
statement when specific values are substituted for the variables.
Example: n divides 8

DEFINITION:
The domain D of a predicate variable is
the set of all values that may be substituted in place of the variable.
The truth set is the set of all elements
of D that make a predicate true.

DEFINITION:
A predicate is a sentence that contains a
finite number of variables and becomes a
statement when specific values are substituted for the variables.
Example: n divides 8

DEFINITION:
The domain D of a predicate variable is
the set of all values that may be substituted in place of the variable.
The truth set is the set of all elements
of D that make a predicate true.
Example: Predicate: n divides 8
Domain : D = Z +
Truth set : {1, 2, 4, 8}

IMPORTANT:
A predicate is not a statement!

IMPORTANT:
A predicate is not a statement!

How To Change Predicates Into


Statements?

IMPORTANT:
A predicate is not a statement!

How To Change Predicates Into


Statements?
I. Assign specific values to variables:
3 divides 8 (False)

IMPORTANT:
A predicate is not a statement!

How To Change Predicates Into


Statements?
I. Assign specific values to variables:
3 divides 8 (False)
II. Add quantifies or :
n Z such that n divides 8(True)
n Z, n divides 8(False)

EXAMPLES OF QUANTIFIED
STATEMENTS:

EXAMPLES OF QUANTIFIED
STATEMENTS:
1. x R, x2 0. (T)

EXAMPLES OF QUANTIFIED
STATEMENTS:
1. x R, x2 0. (T)
2. x R | x2 < 0. (F)

EXAMPLES OF QUANTIFIED
STATEMENTS:
1. x R, x2 0. (T)
2. x R | x2 < 0. (F)
3. x R, if x > 2 then x2 > 4. (T)

EXAMPLES OF QUANTIFIED
STATEMENTS:
1. x R, x2 0. (T)
2. x R | x2 < 0. (F)
3. x R, if x > 2 then x2 > 4. (T)
4. x R | x > 2 and x2 4. (F)

NEGATION RULES
( x D, Q(x)) x D | Q(x)
( x D | Q(x)) x D, Q(x)

Math175 - Discrete Mathematics - Spring 2005


Quiz #7, May 2, 2005
In the following problems you are required to show all your work and provide the necessary explanations everywhere to get full credit.
1. A password consists of 4 digits (each digit can be 0 through 9). How many such passwords
have at least one digit repeated?

2. Suppose three members of a group of 9 insist on working together any team must either
contain all of them or neither. How many distinct 6-person teams can be formed?


 

6:
B
A
A

 

 






11:










 
 A c 
  
  
  
  B 
  
  


















A























































76876:9
?>@?>BA
)<; [.] ,=-/.10
"-2'="354
22: !#"%$&"%'("*)  "+ (.) ,(-/.10
"-2'("*354
(7,5)

(10)

A (A B) = A (A Bc )c = A (Ac B)
(3)

[2]

[1]

= (A Ac ) (A B) = (A B) = (A B)

76876:9
?>@?>BA
)<; [.] ,=-/.10
"-2'="354
24: !#"%$&"%'("*)  "+ (.) ,(-/.10
"-2'("*354
(10)

(3)

(A B) B = (A B) Bc = (A Bc ) (B Bc )
(10)

[2]

[1]

= (A B) (B Bc ) = (A B) = (A B)

76876:9
?>@?>BA
)<; [.] ,=-/.10
"-2'="354
27: !1"*$C"*'("*)  "+ (.) ,(-/.10
"-2'("*354
(10)

(A (A B)) (B (A B)) = (A (A B)c ) (B (A B)c )


(1,2,6)

z }| {
[2]
= = (A B) (A B)c =

35:

 ED

"F+

  
9IH<H 9
OA ED 
)
-G 4
" 'J+1,LKNM  "
"
"F+

SR
9
38: Q
P "+
,T$&-VUWU-FKX+T$Y'(-V3 =, 0<"[Z]\
-2,=M"*)^,`_a1"3 M)<;"*'G.#0
"*-2'("3

9
D

"*U-V)cB+e,=-f-V)
" )<;g-V)<U V
- )<"-2$h,=0<"cdMWid"*)j+k"*,(+
lR

40: ;

)g,=0<"

-8-Vm

,=0

9
,G"  b
0 M)B,("%cd"%'


68
 npo

5:

  A
 9 
H
D

A
H 
L M { qr.r. , .rqr. , .r.rq } s '(MWUMt, 3/8 uMM { .rqXq , qr.rq , qNqr. , qNqNq } s '(- _
9 
D

A
H  9 
D

MUWMW, 4/8 = 1/2 uWM MM { .#.#. } s (' MUMt, 1/8
>@
 nv


14: "

9^

26 10 9 8 = 18720

30:

6! = 720

  

 

5! = 120  3! = 6

 9   9
 9 
 9 
 9
9   9 
9  9
9  9
9   9
33:
s
; s  s  w

; s ; s ;  s
s
;ws  s  ;s ; s ;  s 
s
; s 
w
s
 9
 
 
9  
9 
9 
9
  w



; s ; s ; <
s ;
s<;
s ;
<
s ;

s ;

s ;
<


36: ;
39:

7 6 5 = 210

P(n + 1, 3) P(n, 3) = (n + 1)n(n 1) n(n 1)(n 2)


P(n,2)

z }| {
= ((n + 1) (n 2)) n(n 1) = 3P(n, 2)
x 
 n
10:

 

5! + 5! = 240

9^
9
H
9
H
9
22:
b1000/2c = 500 '("]3y\
UW,=M UW"F+e-2$ 2 s b1000/9c = 111 '="z3y\
Ut,(M U"+1-2$ 9 s
) ;
9
H
 
AJ
H
b1000/18c = 55 '("{3y\
Ut,(M U"+N-2$ 18 uM " -2$ 2 |d}~ 9 .10
"*'="*$C-2'("Vs,=0
"{3y\
UW,=M WU "F+
9

-2$ 2 d 9 '=" 500 + 111 55 = 556
 

556/1000 = 139/250
  1000 556 = 444 

I. Rational Numbers:
1. Z, Q.
2. , Q, + Q.
3. , Q, Q.
4. , Q, Q.
5. , Q, / Q.
6. , Q, if 6= 0 then / Q.
7. , Q, Q.
8. Q, if Z+ then Q.
II. Inequalities:
c
a
< then:
b
d
a
a+c
c
<
< ;
b
b+d
d
at + c
c
a
<
< ;
b
bt + d
d
a
at + c
c
<
< , where t, > 0;
b
bt + d
d
a
at + c
c
<
< , where t, 6= 0;
b
bt + d
d
a
at1 + c1
c
<
< , where t1 , t2 , 1 , 2 > 0.
b
bt2 + d2
d

1. a, b, c, d R+ , if
(a)
(b)
(c)
(d)
(e)

2. a, b, c R+ we have:

(a) a + b 2 ab.
1
(b) a + 2.
a
1
(c) a + 3.
a
a b
(d) + 2.
b a


1 1 1

(e) (a + b + c)
+ +
9.
a b c
(f) a2 + b2 + c2 ab + bc + ca.
(g) ab(a + b) + bc(b + c) + ac(a + c) 6abc.
a b c
+ + 3.
b c a
a
b
c
3
(j)
+
+
.
b+c c+a a+b
2
3
3
3
(k) a + b + c 3abc.
(h)

III. Divisibility:
a, b, c, k Z+ we have:
1. (3k + 1)(3k + 2)(3k + 3) is divisible by 3.
2. If n = 4k + 1, then 8 divides n2 1?
3. If a|b and a|c, then a|(b + c).
4. If a|b and b|a, then a = b or a = b
5. If a b 6= 0, then (a b)|(a2 b2 ).
6 . (a2 + a + 1)|(a3 1).
7 . (a + 1)|(ab + a + b + 1).
8 . (a2 + b2 + ab)|(a4 + a2 b2 + b4 ).
IV. Irrationality:

1. 2 6 Q.

2. 5 + 2 6 Q.

3 . 3 6 Q.

4 . 2 + 3 6 Q.
5 . log2 3 6 Q.

6 . 2 + 3 3 6 Q.

Math175 - Discrete Mathematics - Spring 2005


Quiz #8, May 9, 2005
In the following problems you are required to show all your work and provide the necessary explanations everywhere to get full credit.
1. Let A = {1, 3, 5, 7, 9, . . . , 99}. Prove that if 26 integers are selected from A, then at least one
pair of integers has a sum of 100.


2. Let n and k be positive integers and suppose k n. Prove that

   

n
n
n+1
+
=
.
k1
k
k


 
 14

7: 
= 3432
7  

 
  "! 8 6
#$%! 14
& ''"!
8
=
1400

= 3424
4 3 
7
7

(  14 12 = 2640 
7 
5 
)  12

12
+
= 1584
5
7
  10

10!
= 5!5!
13:
= 252
5
(  10 + 10 = 10 + 1 = 11 
9
10
*  10 + 10 = 1 + 10 = 11 
0
1

20: 
 
( 

8
1

 6
6

8
2

 6
5

8
3

 6
4

= 1016


12!
= 9979200
2!2!1!3!2!1!1!

10!
= 75600
2!2!3!2!1!

9!
= 30240
1!1!1!3!2!1!

+,
 '
 >+@?A$BDC
'E1FGB * )
7: -/.
.10$243%57698;:
0'=< <
n+3
 573 n H


(n + 3)
(n + 3)((n + 3) 1)
=
,
(n + 3) 2
2


3 (n + 3) 2

10: 1, 7, 21, 35, 35, 21, 7, 1

C *
FKB *
J3 57698@:
0
69: %3 576 n = r H
LE
BNMBDC *POQSR F r
M
?AC
 * DB C @
(
* TAQUR  F r+1 = 1 
3J
0
=1
r
r+1
L BDC *
F/B *

V! M
BDC * W
E $BA
'FKB *
3J57698@:
0
69: 3%576 n = k k r
6D: 3J576 n = k + 1 H
3

16: I


 
 

 X

k  
k+2
k+1
k+1
i
k+1
,
=
+
=
+
=
r
+
1
r
+
1
r
r
r
r
i=r
i=r
?AC * * BDC * F *X( E ) *Y
$B 'FBDC * $E ) ( BD E4C
B9C FD'FNM E ) BDC *
FGB E * F
6
5
:1Z0 2
:
5
2,.5 *

0'
5
[ F (
F
^?A$BDC
$E
M E )
$E
!V
_
(
*
_
(
*
k+1
r+1

 Z0]\ 3%57698;:
0'
.10'
573 n 
.10'
573 r

k+1  
X
i

`a
 b

8: 27 33

10
3

= 414720

12: 1.24000 = (1 + 1/5)4000 = 1 + 4000/5 + = 1 + 800 + > 800

c 
 db
Pe PfKhg
 E *i(
2: 
5
5
) q * * ErB ) * E
13
6
5 8
j
 Ss * FNM  BDC * [  g
2
E BD E1FN
 5

4:

8:

) F * *X( B
) C m *
B9C =
gl$m * EnF $ BNMo?AC
 ( Cp?
* ( k6 ) F 7
5 :
0
j
0
Z 0'0
5 3
:
5j:
0

$E BD E F_
 5
) q * * EwB ) * E
M Fu'E (N* BDC *
t
E
 x
* 5 E
C 5j0 * [ 6 'E (  1
. 0*
2vZ6 * 5 0 2 13
6
5j8

) $q * * ErB
s * F_MtFu'E N( * B9C *
E
FDFD   BD * F
DB C ;
* z 6 FuB  E )
2yk6 * 5 02 26 26 = 676
6
. 5
o
0
%3 576
FGB{'E
$BD F
F p
E  F9F
'Eg}BDC * E
F
$ E~BDC *
DB 'E
C  B !V
0'
Z0 7
5 3|@. * 6 5
 :
8
8 * k 6 *
0
Z$0 .  *
e
5

&fKhg
 ( C
F

5,5 * {1, 2, 3, 4, 5}

) ) 'ErB * g * F{$EBDC * gl$m * EvF * BNM B * FGB E *


1
s FNM Fu'E N( * BDC *
E
Eg
2~Z6 * 5 $0 2 n 5
6
 0 
5
Z8}5
11: *
E F  F * B
?A 
m 
E 
 2 :
5 3 n + 1 0$0 ** *
7

16: 81
E !
5 *
18: 16

) DB C *
 'EyBDC * ?
FGB ( F * M
m

E Eax ) $m,Fu  * 
M E ) E DB C *
80 5
576

2l5j:\ * . (9j*
0
2 5 
 5
6

) $ q * * EwB *
) ma'FD E 
M
FD$E (N* DB C *
'E ) * F

2 k6 * 15
~
6
6 8
6 3J576
5
2 15

B $E M,Fu'E _( *
'F
28: * 6  02
366 4 = 1464
  9B C )
B
FGB
F E1FNM,BDC *
6
N
2 573| 8}5
4. * 6 5

)
F9F|BDC E
 m
9B C * *
?
0*
 2000 3 * * u6 2 _2=573
2 k6 * 6 *
) * B
FuB
F
E FV!V
6 *( 5j:
0
 }
8 5
1464 . * 6 5

I. RATIONAL NUMBERS
1. Z, Q. (TRUE)
Proof. We have

.
1
Therefore, since Z, 1 Z, and 1 6= 0, by the definition of rational numbers
we get Q. 
=

2. , Q, + Q. (TRUE)
Proof. We have
=
hence
+ =

p1
,
q1

p2
,
q2

p1 p2
p 1 q2 + p 2 q1
+
=
.
q1
q2
q1 q2

Since p1 , p2 , q1 , q2 Z, it follows that p1 q2 + p2 q1 Z and q1 q2 Z. Plus, since


, Q, it follows that q1 6= 0 and q2 6= 0, therefore q1 q2 6= 0. By this and the
definition of rational numbers we get + Q. 
3. , Q, Q. (TRUE)
Proof. We have
=
hence
=

p1
,
q1

p2
,
q2

p 1 q2 p 2 q 1
p1 p2

=
.
q1
q2
q1 q2

Since p1 , p2 , q1 , q2 Z, it follows that p1 q2 p2 q1 Z and q1 q2 Z. Plus, since


, Q, it follows that q1 6= 0 and q2 6= 0, therefore q1 q2 6= 0. By this and the
definition of rational numbers we get Q. 
4. , Q, Q. (TRUE)
Proof. We have

p1
,
q1

p1 p2
.
q1 q2

=
hence

p2
,
q2

Since p1 , p2 , q1 , q2 Z, it follows that p1 p2 Z and q1 q2 Z. Plus, since , Q,


it follows that q1 6= 0 and q2 6= 0, therefore q1 q2 6= 0. By this and the definition
of rational numbers we get Q. 

5. , Q, / Q. (FALSE)
Counter-Example. Put
= 1,

= 0.

This gives the counter-example, for we cant divide by zero.

6. , Q, if 6= 0 then / Q. (TRUE)
Proof. We have

p1
,
q1

/ =

p 1 q2
.
q1 p 2

=
hence

p2
,
q2

Since p1 , p2 , q1 , q2 Z, it follows that p1 q2 Z and q1 p2 Z. Plus, since Q


and 6= 0, it follows that q1 6= 0 and p2 6= 0, therefore q1 p2 6= 0. By this and the
definition of rational numbers we get / Q. 
7. , Q, Q. (FALSE)
Counter-Example. Put
= 2,

= 1/2,

hence
= 21/2 =

2.

On theone hand, 2 and 1/2 are both rational numbers. On the other
hand, 2 is irrational (see below). This is a contradiction.
8. Q, if Z+ then Q. (TRUE)
Proof. We have
=
hence
=

p1
,
q1

Z+ ,

p1
p1
p
...
= 1 .
q
q
| 1 {z 1} q1

times

Since p1 , q1 Z and Z+ , it follows that p1 Z and q1 Z. Plus, since Q, it


follows that q1 6= 0, therefore q1 6= 0. By this and the definition of rational numbers
we get Q. 

II. INEQUALITIES
1. a, b, c, d R+ , if

<

then:
b
d
a+c
c
a
(a)
<
< ; (TRUE)
b
b+d
d

Proof 1. We have

a
c
< .
b
d
Multiplying both sides of () by bd (note, that bd is positive), we get
ad < bc.

()

()

Adding ab to both sides of (), we get


ab + ad < ab + bc,
which we rewrite by factoring a and b as
a(b + d) < b(a + c).
If we divide both sides of this inequality by b(b+d) (note, that b(b+d) is positive),
we get
a
a+c
<
.
( )
b
b+d
Similarly, adding cd to both sides of (), we get
cd + ad < cd + bc,
which we rewrite by factoring d and c as
d(a + c) < c(b + d).
If we divide both sides of this inequality by d(b+d) (note, that d(b+d) is positive),
we get
a+c
c
< .
( )
b+d
d
It is clear, that ( ) and ( ) give the desired result.
Proof 2 (Short Version Of Proof 1). We have
a
c multiply by bd
+ab
factor a
< ad < bc ab + ad < ab + bc a(b + d) < b(a + c).
bd>0
factor b
b
d
If we divide both sides of the last inequality by b(b + d) (note, that b(b + d) is positive),
we get
a
a+c
<
.
()
b
b+d
Similarly,
a
c multiply by bd
+cd
factor c
< ad < bc ad + cd < bc + cd d(a + c) < c(b + d).
bd>0
factor d
b
d
3

If we divide both sides of the last inequality by d(b + d) (note, that d(b + d) is positive),
we get
a+c
c
< .
()
b+d
d
It is clear, that () and () give the desired result.
Proof 3 (Correct Students Version Of Proof 1). We have
()

a
a + c divide by b(b+d)
factor a
+ab
<
a(b + d) < b(a + c) ab + ad < ab + bc ad < bc.
factor b
b(b+d)>0
b
b+d
The last inequality follows from

a
c
< .
b
d

Similarly,
()

a+c
c divide by d(b+d)
factor c
+cd
< d(a + c) < c(b + d) ad + cd < bc + cd ad < bc.
factor d
d(b+d)>0
b+d
d
The last inequality follows from

a
c
< .
b
d
It is clear, that () and () give the desired result.
Proof 4 (Incorrect Students Version Of Proof 1). We have
a
a + c multiply by b(b+d)
expand
ab
<
a(b + d) < b(a + c) ab + ad < ab + bc ad < bc...
b(b+d)>0
b
b+d

(b)

a
b

<

at + c
bt + d

<

c
d

, where t > 0; (TRUE)

Proof (C.S.V.) We have


()

a
at + c divide by b(bt+d)
factor a
+abt
<
a(bt + d) < b(at + c) abt + ad < abt + bc ad < bc.
factor b
b(bt+d)>0
b
bt + d
The last inequality follows from

a
c
< .
b
d

Similarly,
()

at + c
c divide by d(bt+d)
factor c
+cd
< d(at + c) < c(bt + d) adt + cd < bct + cd adt < bct.
factor d
d(bt+d)>0
bt + d
d
The last inequality follows from

a
c
< .
b
d
It is clear, that () and () give the desired result.

(c)

a
b

<

at + c

<

bt + d

c
d

, where t, > 0; (TRUE)

Proof (C.S.V.) We have


()
at + c divide by b(bt+d)
a
factor a
+abt
<
a(bt+d) < b(at+c) abt+ad < abt+bc ad < bc.
factor
b
b(bt+d)>0
b
bt + d
The last inequality follows from

c
a
< .
b
d

Similarly,
()
c divide by d(bt+d)
at + c
+cd
factor c
< d(at+c) < c(bt+d) adt+cd < bct+cd adt < bct.
factor d
d(bt+d)>0
bt + d
d
The last inequality follows from

a
c
< .
b
d
It is clear, that () and () give the desired result.

(d)

<

at + c

b
bt + d
Counter-Example. Put

<

c
d

, where t, 6= 0; (FALSE)

a = 1, b = 2, c = 1, d = 1, t = 1, = 1.
On the one hand,
a
1
1
c
= < = .
b
2
1
d
On the other hand,
1 1 + 1 (1)
1
1
6<
< .
2
2 1 + 1 (1)
1

(e)

<

at1 + c1

b
bt2 + d2
Counter-Example. Put

<

c
d

, where t1 , t2 , 1 , 2 > 0. (FALSE)

a = 1, b = 2, c = 1, d = 1, t1 = t2 = 1, 1 = 1, 1 = 3.
On the one hand,
a
1
1
c
= < = .
b
2
1
d
On the other hand,

1
11+11
1
6<
< .
2
21+13
1

2. a, b, c R+ we have

(a) a + b 2 ab. (TRUE)


Proof 1 (C.S.V.) We have

root
factor
+4ab
a + b 2 ab (a + b)2 4ab a2 + 2ab + b2 4ab a2 2ab + b2 0.
all

>0

The last inequality is true, since


a2 2ab + b2 = (a b)2 ,
which is always nonnegative. 
Proof 2 (Indirect Proof). Suppose, contrary to our claim, that

a, b R+ | a + b < 2 ab.
We have

square
expand
4ab
a + b < 2 ab (a + b)2 < 4ab a2 + 2ab + b2 < 4ab a2 2ab + b2 < 0.
all

>0

We obtain a contradiction, since the last inequality is false. In fact,


a2 2ab + b2 = (a b)2 ,
which is always nonnegative. 

(b) a +

1
a

2. (TRUE)

Proof (Indirect Proof). Suppose, contrary to our claim, that


a, b R+ | a +

1
< 2.
a

We have

1
a2 + 1
a
< 2
< 2 a2 + 1 < 2a a2 2a + 1 < 0.
a>0
a
a
We obtain a contradiction, since the last inequality is false. In fact,
a+

a2 2a + 1 = (a 1)2 ,
which is always nonnegative. 
1

3. (FALSE)
a
Counter-Example. Put a = 1, then
(c) a +

1+

1
= 2 < 3.
1
6

(d)

a
b

b
a

2. (TRUE)

a
Proof. Put x = , then
b

a b
1
+ =x+ ,
b a
x

which is 2 by (b). 

+ +
9. (TRUE)
a
b
c
Proof. Expanding parentheses, we obtain


1 1 1
a a a b b b c c c
(a + b + c)
+ +
= + + + + + + + +
a b c
a b
c a b c a b c
(e) (a + b + c)

a a b
b c c
+ + +1+ + + +1
b
c a
c a b
 
 

a b
b c
c a
=3+
+
+
+
+
+
.
b a
c b
a c
=1+

Each pair is 2 by (d), therefore



(a + b + c)

1 1 1
+ +
a b c


3 + 2 + 2 + 2 = 9. 

(f ) a2 + b2 + c2 ab + bc + ca. (TRUE)
Proof. By (a) we have
x, y R+ , x2 + y 2 2xy,
therefore
a2 + b2 2ab,

b2 + c2 2bc,

c2 + a2 2ca.

If we add together all the inequalities (), we obtain


2a2 + 2b2 + 2c2 2ab + 2bc + 2ca,
and the result follows. 
(g) ab(a + b) + bc(b + c) + ac(a + c) 6abc. (TRUE)
Proof. We have
ab(a + b) + bc(b + c) + ac(a + c) = a2 b + ab2 + b2 c + bc2 + a2 c + ac2
= (a2 b + bc2 ) + (ab2 + ac2 ) + (b2 c + a2 c)
= b(a2 + c2 ) + a(b2 + c2 ) + c(b2 + a2 ).
By () the right-hand side is b2ac + a2bc + c2ba = 6abc. 
7

()

(h) a3 + b3 + c3 3abc. (TRUE)


Proof. We have
2a3 + 2b3 + 2c3
= (a3 + b3 ) + (b3 + c3 ) + (a3 + c3 )
= (a + b)(a2 ab + b2 ) + (b + c)(b2 bc + c2 ) + (a + c)(a2 ac + c2 ).
= (a + b)(a2 + b2 ab) + (b + c)(b2 + c2 bc) + (a + c)(a2 + c2 ac).
By () the right-hand side is
(a + b)(2ab ab) + (b + c)(2bc bc) + (a + c)(2ac ac)
= (a + b)ab + (b + c)bc + (a + c)ac,
which is 6abc by (g). So, we have
2a3 + 2b3 + 2c3 6abc,
and the result follows. 

(i)

3. (TRUE)
b
c
a
Proof. It follows from (h) that
+

x, y, z R+ , x + y + z 3 3 xyz.
Put

a
x= ,
b

b
y= ,
c

()

c
z= .
a

By this and () we have


r
r
a b c
b c
3 a
3 abc
+ + 3
=3
= 3. 
b c a
b c a
bca

(j)

b+c
Proof. Put

b
c+a

c
a+b

x = b + c,

3
2

. (TRUE)

y = c + a,

z = a + b.

()

From this it follows that


2a = y + z x,

2b = x + z y,
8

2c = x + y z.

()

By () and () we have
2a
2b
2c
+
+
b+c c+a a+b
y+zx x+zy x+yz
=
+
+
x
y
z
y z x x z y x y z
= + + + + +
x x x y y y z z z
y z
x z
x y
= + 1+ + 1+ + 1
x x
y y
z z

 



y x
z y
z x
=
+
+
+
+
+
3,
x y
x z
y z
which is
2+2+23=3
by (d). So,
2a
2b
2c
+
+
3,
b+c c+a a+b
and the result folows. 

III. DIVISIBILITY
a, b, c, k Z+ we have:
1. (3k + 1)(3k + 2)(3k + 3) is divisible by 3.
2. If n = 4k + 1, then 8 divides n2 1.
3. If a|b and a|c, then a|(b + c).
4. Let a, b Z, a 6= 0, b 6= 0. If a|b and b|a, then a = b or a = b.
5. If a b 6= 0, then (a b)|(a2 b2 ).
6 . (a2 + a + 1)|(a3 1).
7 . (a + 1)|(ab + a + b + 1).
8 . (a2 + b2 + ab)|(a4 + a2 b2 + b4 ).
9 . 3 6 | k2 2.
10 . 4 6 | k2 3.
11 . 4 6 | a2 + b2 3.
12 . 8k + 7 6= a2 + b2 + c2 .

IV. IRRATIONALITY
1.

2 6 Q.

2. 5 +
3 .
4 .

2 6 Q.

3 6 Q.
2+

3 6 Q.

5 . log2 3 6 Q.
6 .

2+

3 6 Q.

10

V. EXTRA INEQUALITIES
a, b, c R+ we have:
1.

3
a+b+c

<

1
a+b

1
b+c

1
c+a

2 . a2 (1 + b2 ) + b2 (1 + c2 ) + c2 (1 + a2 ) 6abc.
3 . a2 b2 + b2 c2 + c2 a2 abc(a + b + c).
4 . If a + b 1, then a4 + b4 1/8.

11

Math175 - Discrete Mathematics - Spring 2005


Quiz #9, May 16, 2005
In the following problems you are required to show all your work and provide the necessary explanations everywhere to get full credit.
1. Find all sequences that satisfy the recurrence ak = ak1 + 6ak2 , k 2, and have the form
1, t, t2 , . . . , tn , . . . , where t is nonzero.

2. Use the definition of O-notation to show that 5x9 20x5 + 2x3 x + 15 is O(x9 ).

III. DIVISIBILITY
DEFINITION: If n, d Z and d 6= 0, then n is divisible by d if, and only if, n = d k
for some k Z.
NOTATION: d | n means n is divisible by d or d divides n.
1. k Z+ , (3k + 1)(3k + 2)(3k + 3) is divisible by 3. (TRUE)
Proof. We have
(3k + 1)(3k + 2)(3k + 3) = 3(3k + 1)(3k + 2)(k + 1).

()

Since k Z, it follows that (3k + 1)(3k + 2)(k + 1) Z. This, () and the definition
above give the desired result. 
2. k Z+ , if n = 4k + 1, then 8 divides n2 1. (TRUE)
Proof. We have
n2 1 = (4k + 1)2 1 = 16k 2 + 8k + 1 1 = 16k 2 + 8k = 8(2k 2 + k).

()

Since k Z, it follows that (2k 2 + k) Z. This, () and the definition


above give the desired result. 
3. a, b, c Z+ , if a|b and a|c, then a|(b + c). (TRUE)
Proof. Since a|b and a|c, by the definition above we have
b = a k1 ,

c = a k2

for some k1 , k2 Z. Therefore


b + c = a k1 + a k2 = a(k1 + k2 ).

()

Since k1 , k2 Z, it follows that (k1 + k2 ) Z. We also note that a 6= 0, for a Z+ .


This, () and the definition above give the desired result. 
4. Let a, b Z, a 6= 0, b 6= 0. If a|b and b|a, then a = b or a = b. (TRUE)
Proof. Since a|b and b|a, by the definition above we have
b = ak1 ,

a = bk2

()

for some k1 , k2 Z. Multiplying out these equalities, we obtain


ab = abk1 k2 .
1

()

Since a 6= 0 and b 6= 0, it follows that ab 6= 0, therefore we can cancel out ab from


(). We get
1 = k1 k2 .
From this it follows that k1 = k2 = 1 or k1 = k2 = 1. This and () give the desired
result. 
5. a, b Z, if a b 6= 0, then (a b)|(a2 b2 ). (TRUE)
Proof. We have
a2 b2 = (a b)(a + b).

()

Since a, b Z, it follows that (a b) Z and (a + b) Z. Since a b 6= 0, this, ()


and the definition above give the desired result. 
6 . a Z+ , (a2 + a + 1)|(a3 1). (TRUE)
Proof. We have
a3 1 = (a2 + a + 1)(a 1).

()

Since a Z+ , it follows that (a2 + a + 1) Z+ and (a 1) Z. This, () and the


definition above give the desired result. 
7 . a, b Z+ , (a + 1)|(ab + a + b + 1). (TRUE)
Proof. We have
ab + a + b + 1 = a(b + 1) + b + 1 = (a + 1)(b + 1).

()

Since a, b Z+ , it follows that (a + 1) Z+ and (b + 1) Z+ . This, () and the


definition above give the desired result. 
8 . a, b Z+ , (a2 + b2 + ab)|(a4 + a2 b2 + b4 ). (TRUE)
Proof. We have
a4 + a2 b2 + b4 = (a4 + 2a2 b2 + b4 ) a2 b2 = (a2 + b2 )2 a2 b2
= (a2 + b2 )2 (ab)2

()

= (a2 + b2 + ab)(a2 + b2 ab).


Since a, b Z+ , it follows that (a2 + b2 + ab) Z+ , and (a2 + b2 ab) Z. This, ()
and the definition above give the desired result. 

THEOREM (The Quotient-Remainder Theorem):


For any n Z and d Z+ , there exist unique numbers q, r Z such that
n = d q + r,

where 0 r < d.

9 . k Z, 3 6 | k2 2. (TRUE)
Proof (Indirect). Suppose, contrary to our claim, that
k Z | 3 divides k 2 2.
By the definition above we have
k 2 2 = 3m

()

for some m Z. On the other hand, by the theorem above we have only three possibilities:
k = 3q,

k = 3q + 1,

or k = 3q + 2,

where q Z. Therefore, for k 2 we have only two possibilities:


k 2 = 9q 2 = 3r,
k 2 = (3q + 1)2 = 9q 2 + 6q + 1 = 3(3q 2 + 2q ) + 1 = 3r + 1,
| {z }
r

or
k 2 = (3q + 2)2 = 9q 2 + 12q + 4 = 9q 2 + 12q + 3 + 1 = 3(3q 2 + 4q + 1) + 1 = 3r + 1,
|
{z
}
r

where r Z, for q Z. Consider each of them.


Case I. Let k 2 = 3r. Substituting this into (), we get
3r 2 = 3m,
hence
3r = 3m + 2,
which contradicts the theorem above.
Case II. Let k 2 = 3r + 1. Substituting this into (), we get
3r + 1 2 = 3m,
hence
3r = 3m + 1,
which contradicts the theorem above. 
10 . k Z, 4 6 | k2 3. (TRUE)
3

Proof (Indirect). Suppose, contrary to our claim, that


k Z | 4 divides k 2 3.
By the definition above we have
k 2 3 = 4m

()

for some m Z. On the other hand, by the theorem above we have only two possibilities:
k = 2q

or k = 2q + 1,

where q Z. Therefore, for k 2 we have also only two possibilities:


k 2 = 4q 2 = 4r
or
k 2 = 4q 2 + 4q + 1 = 4(q 2 + q ) + 1 = 4r + 1,
| {z }
r

where r Z, for q Z. Consider each of them.


Case I. Let k 2 = 4r. Substituting this into (), we get
4r 3 = 4m,
hence
4r = 4m + 3,
which contradicts the theorem above.
Case II. Let k 2 = 4r + 1. Substituting this into (), we get
4r + 1 3 = 4m,
hence
4r = 4m + 2,
which contradicts the theorem above. 
11 . a, b Z, 4 6 | a2 + b2 3. (TRUE)
Proof (Indirect). Suppose, contrary to our claim, that
a, b Z | 4 divides a2 + b2 3.
By the definition above we have
a2 + b2 3 = 4m
for some m Z. On the other hand, by the theorem above for any k Z we have only
two possibilities:
k = 2q or k = 2q + 1,

()

where q Z. Therefore, for k 2 we have also only two possibilities:


k 2 = 4q 2 = 4r
or
k 2 = 4q 2 + 4q + 1 = 4(q 2 + q ) + 1 = 4r + 1,
| {z }
r

where r Z, for q Z. From this it follows that for a2 + b2 3 we have only three
possibilities:
a2 + b2 3 = 4r1 + 4r2 3 = 4(r1 + r2 ) 3 = 4R 3,
| {z }
R

a2 + b2 3 = 4r1 + 4r2 + 1 3 = 4(r1 + r2 ) 2 = 4R 2,


| {z }
R

or
a2 + b2 3 = 4r1 + 4r2 + 2 3 = 4(r1 + r2 ) 1 = 4R 1,
| {z }
R

where R Z, for r1 , r2 Z. If we compare each of them with (), we obtain contradictions


as in the proofs above. 
12 . a, b, c Z and k Z+ , 8k + 7 6= a2 + b2 + c2 . (TRUE)
Proof (Indirect). Suppose, contrary to our claim, that
a, b Z and k Z+ | 8k + 7 = a2 + b2 + c2 .

()

On the other hand, by the theorem above for any k Z we have only eight possibilities:
k = 8q,

k = 8q + 1, . . . , k = 8q + 6,

or k = 8q + 7,

where q Z. From this one can deduce that for k 2 we have only three possibilities:
k 2 = 8r,

k 2 = 8r + 1,

or k 2 = 8r + 4,

where r Z. From this it follows that there is no combination which gives


a2 + b2 + c2 = 4R + 7.
We obtain a contradiction. 

Math175 - Discrete Mathematics - Spring 2005


Quiz #10, May 23, 2005
In the following problems you are required to show all your work and provide the necessary explanations everywhere to get full credit.
1. Determine which of the following graphs have Euler circuits. Find Euler circuits for those
graphs that have them.

2. For each of the following graphs, determine whether there is an Euler path from u to w. If
there is, find such a path.

THEOREM (Some Subset Relations):


1. Inclusion of Intersection: For all sets A and B,
(a) A B A and (b) A B B.
2. Inclusion in Union: For all sets A and B,
(a) A A B

and (b) B A B.

3. Transitive Property of Subsets: For all sets A, B, and C,


if A B

and B C,

then A C.

Proof:
1(a). Suppose A and B are any sets and suppose x is any element of A B. Then x A
and x B by definition of intersection. So, x A.
1(b). Suppose
and
are any sets and suppose
and
by definition of intersection. So,
.

is any element of

. Then

2(a). Suppose
by definition of

and

are any sets and suppose


.

is any element of

. Then

2(b). Suppose
by definition of

and

are any sets and suppose


.

is any element of

. Then

3. Suppose A, B, and C are any sets and suppose A B and B C. To show that A C,
we must show that every element in A is in C. To this end we note that if x A, then
x B (because A B) and therefore x C (because B C). Hence A C. 

EXAMPLES:
1. Prove that for all sets A and B,
A B A.
Proof:
Suppose
and
are any sets and suppose
by definition of
. So,
.

is any element of

. Then

and

2. Prove that for all sets A, B, and C,


A (B C) = (A B) (A C).
Proof:
Suppose , , and
are any sets.
(I). We first prove that A (B C) (A B) (A C). Let x A (B C). By definition
of intersection, x
and x
. Thus x A and by definition of union, x B or
.
Case 1 (x A and x B): In this case, by definition of intersection x
, and so by
definition of union, x (A B) (A C).
Case 2 (x A and x C): In this case, by definition of intersection x
, and so by
.
definition of union,
Hence in either case, x (A B) (A C).
(II). We now prove that (A B) (A C) A (B C). Let x (A B) (A C). By
definition of union, x
or x
.
Case 1 (x A B): In this case, by definition of intersection
and
. Since
x B, by definition of union, x B C. Hence x A and x B C, and so by definition
of intersection, x
.
and
. Since
Case 2 (x A C): In this case, by definition of intersection
, by definition of union,
. Hence
and
, and so by definition
.
of intersection, x
In either case x A (B C). 

THEOREM (Set Identities):


Let all sets referred to below be subsets of a universal set U.
1. Commutative Laws: For all sets A and B,
(a) A B = B A and (b) A B = B A.
2. Associative Laws: For all sets A, B, and C,
(a) (A B) C = A (B C) and (b) (A B) C = A (B C).
3. Distributive Laws: For all sets A, B, and C,
(a) A (B C) = (A B) (A C) and (b) A (B C) = (A B) (A C).
4. Intersection with U : For all sets A,
A U = A.
5. Double Complement Law: For all sets A,
(Ac )c = A.
6. Idempotent Laws: For all sets A,
(a) A A = A and (b) A A = A.
7. De Morgans Laws: For all sets A and B,
(a) (A B)c = Ac B c

and (b) (A B)c = Ac B c .

8. Union with U : For all sets A,


A U = U.
9. Absorption Laws: For all sets A and B,
(a) A (A B) = A and (b) A (A B) = A.
10. Alternate Representation for Set Difference: For all sets A and B,
A B = A Bc.

EXERCISE SET:
For all sets A, B, and C,
1. (A B) (A B) = A.
2. (A B) C = (A C) (B C).
3. (A B) (C A) = A (B C).
4. (A B) (B C) = A B.
5. (A B) (B A) = (A B) (A B).
6. [(Ac B c ) A]c = A.

THEOREM (Some Subset Relations):


1. Inclusion of Intersection: For all sets A and B,
(a) A B A and (b) A B B.
2. Inclusion in Union: For all sets A and B,
(a) A A B

and (b) B A B.

3. Transitive Property of Subsets: For all sets A, B, and C,


if A B

and B C,

then A C.

Proof:
1(a). Suppose A and B are any sets and suppose x is any element of A B. Then x A
and x B by definition of intersection. So, x A.
1(b). Suppose A and B are any sets and suppose x is any element of A B. Then x A
and x B by definition of intersection. So, x B.
2(a). Suppose A and B are any sets and suppose x is any element of A. Then x A B
by definition of union.
2(b). Suppose A and B are any sets and suppose x is any element of B. Then x A B
by definition of union.
3. Suppose A, B, and C are any sets and suppose A B and B C. To show that A C,
we must show that every element in A is in C. To this end we note that if x A, then
x B (because A B) and therefore x C (because B C). Hence A C.

SOLUTIONS
1. (A B) (A B) = A.
Solution. We have:
(10)

(A B) (A B) = (A B c ) (A B)
(3)

= A (B c B)

def.

= AU

(4)

= A.

2. (A B) C = (A C) (B C).
Solution. We have:
(1)

(A B) C = C (A B)
(3)

= (C A) (C B)

(1)

= (A C) (B C).

3. (A B) (C A) = A (B C).
Solution. We have:
(10)

(A B) (C A) = (A B) (C Ac )
(10)

= (A B) (C Ac )c

(7)

= (A B) (C c (Ac )c )

(5)

= (A B) (C c A)

(1)

= (A B) (A C c )

(3)

= A (B C c )

(10)

= A (B C).

4. (A B) (B C) = A B.
Solution. We have:
(10)

(A B) (B C) = (A B c ) (B C c )
(10)

= (A B c ) (B C c )c

(2)

= A [B c (B C c )c ]

(7)

= A [B (B C c )]c

(9)

= A Bc

(10)

= A B.
1

5. (A B) (B A) = (A B) (A B).
Solution. We have:
(10)

(A B) (B A) = (A B c ) (B Ac )
(3)

= [(A B c ) B] [(A B c ) Ac ]

(1)

= [B (A B c )] [Ac (A B c )]

(3)

= [(B A) (B B c )] [(Ac A) (Ac B c )]

def.

= [(B A) U ] [U (Ac B c )]

(1)

= [(B A) U ] [(Ac B c ) U ]

(4)

= (B A) (Ac B c )

(7)

= (B A) (A B)c

(1)

= (A B) (A B)c

(10)

= (A B) (A B).

6. [(Ac B c ) A]c = A.
Solution. We have:
(7)

[(Ac B c ) A]c = [(A B)c A]c


(10)

= [(A B)c Ac ]c

(7)

= ((A B)c )c (Ac )c

(5)

= (A B) A

(1)

= A (A B)

(9)

= A.

EXERCISES:
1. We toss three coins.
(a) Write the sample space of possible outcomes.
(b) What is the probability that 0 heads are obtained?
(c) What is the probability that 1 head is obtained?
(d) What is the probability that 2 heads are obtained?
(e) What is the probability that 3 heads are obtained?
2. We toss a die.
(a) Write the sample space of possible outcomes.
(b) What is the probability that the number showing face up is 1?
(c) What is the probability that the number showing face up is even?
(d) What is the probability that the number showing face up is prime?
3. We toss two dice.
(a) How many possible outcomes are there?
(b) What is the probability that the numbers showing face up are equal?
(c) What is the probability that the numbers showing face up have a sum 6?
(d) What is the probability that the numbers showing face up are different?

COUNTING ELEMENTS OF SETS

NOTATION: For any finite set, n(A) denotes the number of elements in A.

THEOREM 1 (The Addition Rule):


Suppose a finite set A equals the union of k distinct mutually disjoint subsets A1 , A2 , . . . , Ak .
Then
n(A) = n(A1 ) + n(A2 ) + . . . + n(Ak ).
THEOREM 2 (The Difference Rule):
If A is a finite set and B is a subset of A, then
n(A B) = n(A) n(B).

THEOREM 3 (The Inclusion/Exclusion Rule):


If A, B, and C are any finite sets, then
n(A B) = n(A) + n(B) n(A B)
and
n(A B C)
= n(A) + n(B) + n(C) n(A B) n(A C) n(B C) + n(A B C).

EXAMPLES:
1. A code word consists of from one to three letters chosen from the 26 in the alphabet with
repetitions allowed. How many different code words are possible?
Solution: By Theorem 1, the total number of code words equals the
number of code words of length 1 (which = 26)
+
number of code words of length 2 (which = 262 )
+
number of code words of length 3 (which = 263 ).
Hence, the total number of code words
= 26 + 262 + 263 = 18, 278. 

2. How many integers from 1 through 999 do not have any repeated digits?
Solution: By Theorem 1, the number of integers from 1 through 999 with no repeated digits
equals the
number of integers from 1 through 9 with no repeated digits (which = 9)
+
number of integers from 10 through 99 with no repeated digits (which = 9 9)
+
number of integers from 100 through 999 with no repeated digits (which = 9 9 8).
Hence, the total number of integers from 1 through 999 with no repeated digits
= 9 + 9 9 + 9 9 8 = 738. 

3. How many integers from 1 through 999 have at least one repeated digit?
Solution: By Theorem 2, the number of integers from 1 through 999 with at least one
repeated digit equals the
total number integers from 1 through 999 (which = 999)
number of integers from 1 through 999 with no repeated digits (which = 738).
Hence, the total number of integers from 1 through 999 with at least one repeated digit
= 999 738 = 261. 

4. How many integers from 1 through 1,000 are multiples of 3 or multiples of 5?


Solution: We will use Theorem 3. Let
A = the set of all integers from 1 through 1,000 that are multiples of 3
B = the set of all integers from 1 through 1,000 that are multiples of 5.
Then
A B = the set of all integers from 1 through 1,000 that are multiples of 3 or 5
A B = the set of all integers from 1 through 1,000 that are multiples of 3 and 5
= the set of all integers from 1 through 1,000 that are multiples of 15.
By Theorem 3,
n(A B) = n(A) + n(B) n(A B).
So, to solve this problem we should find n(A), n(B), and n(A B). One can see that
n(A):
there are [1, 000/3] = 333 numbers from 1 through 1,000 that are multiples of 3
n(B):
there are [1, 000/5] = 200 numbers from 1 through 1,000 that are multiples of 5
n(AB): there are [1, 000/15] = 66 numbers from 1 through 1,000 that are multiples of 15.
Hence by Theorem 3, the total number of integers from 1 through 1,000 that are multiples
of 3 or multiples of 5
= 333 + 200 66 = 467. 
2

EXERCISES:
1. How many arrangements of no more that three letters can be formed using the letters
of the word N ET W ORK with
(a) repetitions allowed?
(b) no repetitions allowed?
2. (a) How many ways can the letters of the word QU ICK be arranged in a row if the Q
and the U must remain next to each other in the order QU ?
(b) How many ways can the letters of the word QU ICK be arranged in a row if the
letters QU must remain together but may be in either the order order QU or the
order U Q?
3. A group of eight people are attending the movies together.
(a) Two of the eight insist on sitting together. In how many ways can the eight be seated
in a row?
(b) Two of the people do not like each other and do not want to sit side-by-side. Now
how many ways can the eight be seated in a row?
4. (a) Assuming that any ten digits can be used to form a telephone number, how many
seven-digit telephone numbers do not have any repeated digits?
(b) How many seven-digit telephone numbers have at least one repeated digit?
(c) What is the probability that a randomly chosen seven-digit telephone number has
at least one repeated digit?
5. How many integers from 1 through 100,000 contain the digit 3 exactly once?
6. How many integers from 1 through 1,000 are multiples of 4 or multiples of 7?
7 . Suppose a public opinion polltaker reports that out of a national sample of 1,200 adults
(i)

675 are married;

(ii)

682 are from 20 to 30 years old;

(iii)

684 are female;

(iv)

195 are married and are from 20 to 30 years old;

(v)

467 are married females;

(vi)

318 are females from 20 to 30 years old;

(vii) 165 are married females from 20 to 30 years old.


Are the polltakers figures consistent?
8 . How many positive integers less than 1,000 have no common factors with 1,000?

COUNTING SUBSETS OF A SET: COMBINATIONS


DEFINITION 1:
Let n, r be nonnegative integers with r n. An r-combination of a set of n elements is a
subset of r of the n elements.
EXAMPLE 1: Let S = {a, b, c, d}. Then
the 1-combinations are : {a}, {b}, {c}, {d}
the 2-combinations are : {a, b}, {a, c}, {a, d}, {b, c}, {b, d}, {c, d}
the 3-combinations are : {a, b, c}, {a, b, d}, {a, c, d}, {b, c, d}
the 4-combination is : {a, b, c, d}.

DEFINITION 2:
 
n
The symbol
, read n choose r, denotes the number of r-combinations that can be
r
chosen from a set of n elements.

EXAMPLE 2: It follows from Example 1 that


 
 
 
4
4
4
= 4,
= 6,
= 4,
1
2
3

 
4
= 1.
4

THEOREM: Let n, r be nonnegative integers with r n. Then


 
n
n!
=
.
r
r! (n r)!

EXAMPLE 3: We have
 
4!
4!
4
=
=
2
2! (4 2)!
2! 2!
 
4
4!
4!
=
=
3
3! (4 3)!
3! 1!
 
4
4!
4!
=
=
4
4! (4 4)!
4! 0!
 
8!
8
8!
=
=
5
5! (8 5)!
5! 3!

1234
34
=
= 6,
(1 2) (1 2)
12

1234
4
= = 4,
(1 2 3) 1
1

1234
1
= = 1,
(1 2 3 4) 1
1

12345678
678
78
=
=
= 56.
(1 2 3 4 5) (1 2 3)
123
1

PROBLEMS:
1. Suppose 5 members of a group of 12 are to be chosen to work as a team on a special
project. How many distinct 5-person teams can be formed?
2. Suppose two members of the group of 12 insist on working as a pair any team must
either contain both or neither. How many distinct 5-person teams can be formed?
3. Suppose two members of the group of 12 refuse to work together on a team. How many
distinct 5-person teams can be formed?
4. Suppose the group of 12 consists of 5 men and 7 women.
(a) How many 5-person teams can be chosen that consist of 3 men and 2 women?
(b) How many 5-person teams contain at least one man?
(c) How many 5-person teams contain at most one man?
5. Consider various ways of ordering the letters in the word MISSISSIPPI:
IIMSSPISSIP, ISSSPMIIPIS, PIMISSSSIIP,
How many distinguishable orderings are there?

and so on.

SOLUTIONS:
1. Suppose 5 members of a group of 12 are to be chosen to work as a team on a special
project. How many distinct 5-person teams can be formed?
Solution: The number of distinct 5-person teams is the same as the number of subsets of
size 5 (or 5-combinations) that can be chosen from the set of 12. This number is
 
12
12!
12!
1 2 3 4 5 6 7 8 9 10 11 12
=
=
=
5
5! (12 5)!
5! 7!
(1 2 3 4 5) (1 2 3 4 5 6 7)
=

8 9 10 11 12
9 10 11 12
=
= 3 2 11 12 = 792. 
12345
135

2. Suppose two members of the group of 12 insist on working as a pair any team must
either contain both or neither. How many distinct 5-person teams can be formed?
Solution: Call the two members of the group that insist on working as a pair A and B.
Then any team formed must contain both A and B or neither A nor B. By Theorem 1 (The
Addition Rule) we have:

number of 5-person teams


number of 5-person
number of 5-person

containing both A and B = teams containing


+ teams containing
.
or neither A nor B
both A and B
neither A nor B
Because a team that contains both A and B contains exactly 3 other people from the
remaining 10 in the group, there are as many such teams as there are subsets of 3 people that
can be chosen from the remaining 10. This number is
 
10
10!
10!
1 2 3 4 5 6 7 8 9 10
8 9 10
=
=
=
= 4 3 10 = 120.
=
3
3! (10 3)!
3! 7!
(1 2 3) (1 2 3 4 5 6 7)
123
Similarly, because a team that contains neither A nor B contains exactly 5 people from the
remaining 10 in the group, there are as many such teams as there are subsets of 5 people that
can be chosen from the remaining 10. This number is
 
10
10!
10!
1 2 3 4 5 6 7 8 9 10
6 7 8 9 10
=
=
=
=
= 252.
5
5! (10 5)!
5! 5!
(1 2 3 4 5) (1 2 3 4 5)
12345
Therefore,

number of 5-person teams

containing both A and B = 120 + 252 = 372. 


or neither A nor B

3. Suppose two members of the group of 12 refuse to work together on a team. How many
distinct 5-person teams can be formed?
Solution: Call the two members of the group that refuse to work together A and B. By
Theorem 2 (The Difference Rule) we have:

number of 5-person teams


number of 5-person teams


total number of

that contain

=
that dont contain
5-person teams
both A and B
both A and B
   
12
10
=

= 792 120 = 672. 


5
3
4. Suppose the group of 12 consists of 5 men and 7 women.
(a) How many 5-person teams can be chosen that consist of 3 men and 2 women?
(b) How many 5-person teams contain at least one man?
(c) How many 5-person teams contain at most one man?
Solution:
 
 
5
7
(a) Note, that there are
ways to choose the three men out of the five and
ways
3
2
to choose the two women out of the seven. Therefore, by The Multiplication Rule we have:
    

number of 5-person teams that
5
7
=

= 210.
3
2
contain 3 men and 2 women

(b) By Theorem 2 (The Difference Rule) we have:


 
 

number of 5-person teams
total number
number of 5-person teams
=

.
with at least one man
of 5-person teams
that do not contain any men

Now a 5-person team with


  no men consists of 5 women chosen from the seven women in the
7
group. So, there are
= 21 such teams. Also, the total number of 5-person teams is
5
 
12
= 792. Therefore,
5


number of 5-person teams
= 792 21 = 771.
with at least one man

(c) By Theorem 1 (The Addition Rule) we have:


 

 
number of 5-person teams
number of 5-person teams
number of 5-person teams
.
=
+
with at most one man
without any men
with one man

Now a 5-person team without


  any men consists of 5 women chosen from the 7 women in the
7
group. So, there are
= 21 such teams. Also, by The Multiplication Rule there are
5
   
5
7

= 175 teams with one man. Therefore,


1
4


number of 5-person teams
= 21 + 175 = 196. 
with at most one man
4

5. Consider various ways of ordering the letters in the word MISSISSIPPI:


IIMSSPISSIP, ISSSPMIIPIS, PIMISSSSIIP,

and so on.

How many distinguishable orderings are there?


Solution: Since there are 11 positions in all, there are
 
11
subsets of 4 positions for the Ss.
4
Once the four Ss are in place, there are
 
7
subsets of 4 positions for the Is.
4
After the Is are in place, there are
 
3
subsets of 2 positions for the P s.
2
That leaves just one position for the M . Hence, by The Multiplication Rule we have:

      
number of ways to
11
7
3
=

= 34, 650. 
4
4
2
position all the letters

THE ALGEBRA OF COMBINATIONS

EXERCISE SET:
1. For all integers n 2 we have



n
n(n 1)
=
.
2
n2

2. For all integers n, k with n k + 1 1 we have




 
n
nk n
=
.
k+1
k+1 k
3 . For all integers n, k with n k 1 we have


 

  
n
n
n
n+2
+2
+
=
.
k1
k
k+1
k+1
4 . For all integers n, k with n k 1 we have

2  2 
2
 

n+1
n
n
n
n

=2
.
k
k
k1
k
k1
5 . For all integers n 1 we have
   

 

2
3
n+1
n+2
+
+ ... +
=
.
2
2
2
3
6 . Are there integer numbers n, k such that
 


n
n
= 1001,
= 2002,
k
k+1


n
= 3003?
k+2

THE BINOMIAL THEOREM

EXERCISE SET:
1 . For all integers n 1 we have
     
 
n
n
n
n n

+
. . . + (1)
= 0.
0
1
2
n
2 . Let n 0 be an integer number. Find
     
n
n
n
+
+
+ ...
0
2
4
3 . Let n 1 be an integer number. Find
     
n
n
n
+
+
+ ...
1
3
5
4 . For all integers n 0 we have
 
 
 
 
n
n
2 n
n n
+2
+2
+ ... + 2
= 3n .
0
1
2
n
5 . For all integers n 0 we have
 2  
 2  2   2
n
2n
n
n
n
=
.
+
+ ... +
+
n
n
1
2
0

6 . For all integers n 1 we have


 
 
 
 
n
n
n
n
+2
+3
+ ... + n
= n 2n1 .
1
2
3
n

(x+1)0 = 1
(x+1)1 = x+1
(x+1)2 = x2 +2x+1
(x+1)3 = x3 +3x2 +3x+1
(x+1)4 = x4 +4x3 +6x2 +4x+1
(x+1)5 = x5 +5x4 +10x3 +10x2 +5x+1
(x+1)6 = x6 +6x5 +15x4 +20x3 +15x2 +6x+1
(x+1)7 = x7 +7x6 +21x5 +35x4 +35x3 +21x2 +7x+1
(x+1)8 = x8 +8x7 +28x6 +56x5 +70x4 +56x3 +28x2 +8x+1
(x+1)9 = x9 +9x8 +36x7 +84x6 +126x5 +126x4 +84x3 +36x2 +9x+1

(x+1)0 = 1
(x+1)1 = x+1
(x+1)2 = x2 +2x+1
(x+1)3 = x3 +3x2 +3x+1
(x+1)4 = x4 +4x3 +6x2 +4x+1
(x+1)5 = x5 +5x4 +10x3 +10x2 +5x+1
(x+1)6 = x6 +6x5 +15x4 +20x3 +15x2 +6x+1
(x+1)7 = x7 +7x6 +21x5 +35x4 +35x3 +21x2 +7x+1
(x+1)8 = x8 +8x7 +28x6 +56x5 +70x4 +56x3 +28x2 +8x+1
(x+1)9 = x9 +9x8 +36x7 +84x6 +126x5 +126x4 +84x3 +36x2 +9x+1
(x+1)10 = x10 +10x9 +45x8 +120x7 +210x6 +252x5 +210x4 +120x3 +45x2 +10x+1
(x+1)11 = x11 +11x10 +55x9 +165x8 +330x7 +462x6 +462x5 +330x4 +165x3 +55x2 +11x+1
(x+1)12 = x12 +12x11 +66x10 +220x9 +495x8 +792x7 +924x6 +792x5 +495x4 +220x3 +66x2 +12x+1
(x+1)13 = x13 +13x12 +78x11 +286x10 +715x9 +1287x8 +1716x7 +1716x6 +1287x5 +715x4 +286x3 +78x2 +13x+1
(x+1)14 = x14 +14x13 +91x12 +364x11 +1001x10 +2002x9 +3003x8 +3432x7 +3003x6 +2002x5 +1001x4 +364x3 +91x2 +14x+1
(x+1)15 = x15 +15x14 +105x13 +455x12 +1365x11 +3003x10 +5005x9 +6435x8 +6435x7 +5005x6 +3003x5 +1365x4 +455x3 +105x2 +15x+1
(x+1)16 = x16 +16x15 +120x14 +560x13 +1820x12 +4368x11 +8008x10 +11440x9 +12870x8 +11440x7 +8008x6 +4368x5 +1820x4 +560x3 +120x2 +16x+1
(x+1)17 = x17 +17x16 +136x15 +680x14 +2380x13 +6188x12 +12376x11 +19448x10 +24310x9 +24310x8 +19448x7 +12376x6 +6188x5 +2380x4 +680x3 +136x2 +17x+1
(x+1)18 = x18 +18x17 +153x16 +816x15 +3060x14 +8568x13 +18564x12 +31824x11 +43758x10 +48620x9 +43758x8 +31824x7 +18564x6 +8568x5 +3060x4 +816x3 +153x2 +18x+1

The Pigeonhole Principle

PRINCIPLE:
If we put N + 1 or more pigeons into N pigeon holes, then at least one pigeon hole will
contain two or more pigeons.
FIRST SIMPLE EXAMPLES:
1. Among any group of 367 people, there must be at least two with the same birthday, because
there are only 366 possible birthdays.
2. Among a group of 11 people in the elevator of a 10 story building, there must be at least
two who will exit the elevator on the same floor.
3. One million trees grow in a forest. It is known that no tree has more than 600,000 leaves.
Show that at any moment there are at least two trees in the forest that have exactly the
same number of leaves.
Solution: Trees are pigeons and numbers of their leaves are pigeon holes. As there are more
pigeons than pigeon holes, there will be a pigeon hole with more than one pigeon
in it.

FURTHER EXAMPLES:
1. 12 students wrote a dictation. John Smart made 10 errors, each of the other students
made less than that number. Prove that at least two students made equal number of
errors.
Solution: Let us pretend that the students are pigeons and put them in 11 holes numbered
0, 1, 2, ... , 10, according to the number of errors made. In hole 0 we put those
students who made no errors, in hole 1 those who made exactly 1 error, in hole 2
those who made 2 errors, and so on. Certainly, hole 10 is occupied solely by John
Smart. Now apply the Pigeonhole Principle.

2. How many cards must be selected from a standard deck of 52 cards to ensure that we get
at least 3 cards of the same suit?
Solution: Since there are 4 suits, if we only select 8 cards then it is possible that we get 2
cards of each suit. So 8 is not enough to guarantee at least 3 cards of the same
suit. However, if we select 9 cards then the Pigeonhole Principle tells us that we
will get at least 9/4 = 3 cards of the same suit. So 9 is the least we can select to
guarantee at least 3 cards of the same suit.

3. Let A = {1, 2, 3, 4, 5, 6, 7, 8}. Prove that if five integers are selected from A, then at least
one pair of integers have a sum of 9.
Solution: Partition the set A into 4 subsets:
{1, 8},

{2, 7},

{3, 6},

and {4, 5},

each consisting of two integers whose sum is 9. If 5 integers are selected from A,
then by the Pigeonhole Principle at least two must be from the same subset. But
then the sum of these two integers is 9.

4. A plane is colored blue and red. Is it always possible to find two points of the same color
exactly 1 inch apart?
Solution: Think of an equilateral triangle with each side exactly 1 inch long. At least two
of its vertices have to be of the same color (colors are pigeon holes, and vertices
of the triangle are pigeons). This proves that there have to be two points of the
same color exactly 1 inch apart.

5. 51 points were placed, in an arbitrary way, into the square of side 1. Prove that some 3
of these points can be covered by a circle of radius 1/7 .
Solution: Divide the square into 25 smaller squares of side 1/5 each. Then at least one of
these small squares - holes- would contain at least three pigeons - points. Indeed,
if this is not true, then every small square contains 2 points or less; but the the
total number of points is no more that 2 25 = 50. This contradicts to the assumption that we have 51 points.
Now the circle circumvented around the square with the three points inside also
contains these three points and has radius
s 
r
r
 2 r
2
1
1
1
2
1
1
r=
+
=
=
<
= .
10
10
100
50
49
7

PROBLEMS:
1. Population of Greater Manchester is above 6,000,000 people, and each has no more than
100,000 hairs on his or her head. Prove that some 60 residents of Greater Manchester
have equal number of hairs.
2. There are 30 classes and 1000 students in the school. Prove that at least one class has
at least 34 students.
3. A group of 25 students wrote a dictation. John Smart made 10 errors, and each of the
2

rest made less than 10 errors. Prove that at least 3 students made equal number of errors.
4. Prove that, given any 12 natural numbers, we can chose two of them and such that their
difference is divisible by 11.
5 . 5 points are positioned inside of the equilateral triangle of side 2. Prove that there are
two of them at the distance less than 1 from each other.
6 . Prove that of any 52 natural numbers one can find two numbers m and n such that
either their sum m + n or their difference m n is divisible by 100. Is the same
statement true for 51 arbitrary natural numbers?
7 . Prove that some integral power of 2 has the decimal expansion which starts with the
digits 1999:
2n = 1999...

3. A group of 25 students wrote a dictation. John Smart made 10 errors, and each of the
rest made less than 10 errors. Prove that at least 3 students made equal number of errors.
Solution: Assume that no three students made equal number of errors. It means that each
of 10 holes 0, 1, 2, 3, ... , 9 contains less than three students. Therefore all these
holes together contain less than 10 2 = 20 students. Add John Smart to this
number, and we get only 21 students, not 25 as given in the problem. We reached
a contradiction.

4. Prove that, given any 12 natural numbers, we can chose two of them and such that their
difference is divisible by 11.
Solution: There are 11 possible remainders upon division by 11:
0, 1, 2, 3, . . . , 10.
But we have 12 numbers. if we take the remainders for holes and the numbers
for pigeons then by the Pigeonhole Principle there are at least two pigeons sharing
the same hole, i.e. two numbers with the same remainder. The difference of these
two numbers is divisible by 11.

GRAPH THEORY: INTRODUCTION


DEFINITION 1:
A graph G consists of two finite sets: a set V (G) of vertices and a set E(G) of edges, where
each edge is associated with a set consisting of either one or two vertices called its endpoints.
The correspondence from edges to endpoints is called the edge-endpoint function. An edge
with just one endpoint is called a loop, and two distinct edges with the same set of endpoints
are said to be parallel. An edge is said to connect its endpoints; two vertices that are
connected by an edge are called adjacent; and a vertex that is an endpoint of a loop is said to
be adjacent to itself. An edge is said to be incident on each of its endpoints, and two edges
incident on the same endpoint are called adjacent. A vertex on which no edges are incident is
called isolated. A graph with no vertices is called empty, and one with at least one vertex is
called nonempty.
EXAMPLE:
Consider the following graph:

(a) Write the vertex set and the edge set, and give a table showing the edge-endpoint function;
(b) Find all that are incident on v1 , all vertices that are adjacent to v1 , all edges that are
adjacent to e1 , all loops, all edges, all vertices that are adjacent to themselves, all isolated
vertices.
Solution:
(a) We have: vertex set = {v1 , v2 , v3 , v4 , v5 , v6 }
edge set = {e1 , e2 , e3 , e4 , e5 , e6 , e7 }
edge-endpoint function :
Edges
e1
e2
e3
e4
e5
e6
e7

Endpoints
{v1 , v2 }
{v1 , v3 }
{v1 , v3 }
{v2 , v3 }
{v5 , v6 }
{v5 }
{v6 }

(b) We have:

, and
and

are incident on v1 .

are adjacent to v1 .

, and

are adjacent to e1 .

and

are loops.

and

are parallel.

and

are adjacent to themselves.

is an isolated vertex.
DEFINITION 2:
A simple graph is a graph that does not have any loops or parallel edges. In a simple graph,
an edge with endpoints v and w is denoted {v, w}.
EXAMPLE:
Draw all simple graphs with the four vertices {u, v, w, x} and two edges, one of which is {u, v}.
Solution:
There are 5 such graphs:

DEFINITION 3:
A complete graph on n vertices, denoted Kn , is a simple graph with n vertices v1 , v2 , . . . , vn
whose set of edges contains exactly one edge for each pair of distinct vertices.
EXAMPLE:
Draw the complete graphs K2 , K3 , K4 , and K5 .
Solution:

DEFINITION 4:
A complete bipartite graph on (m, n) vertices, denoted Km,n , is a simple graph with
vertices v1 , v2 , . . . , vm and w1 , w2 , . . . , wn that satisfies the following properties:
for all i, k = 1, 2, . . . , m and all j, l = 1, 2, . . . , n,
1. There is an edge from each vertex vi to each vertex wj ;
2. There is not an edge from any vertex vi to any other vertex vk ;
3. There is not an edge from any vertex wj to any other vertex wl .
EXAMPLE:
Draw the bipartite graphs K3,2 and K3,3 .
Solution:

DEFINITION 5:
A graph H is said to be a subgraph of a graph G if, and only if, every vertex in H is also a
vertex in G, every edge in H is also an edge in G, and every edge in H has the same endpoints
as in G.
EXAMPLE:
List all nonempty subgraphs of the graph G with vertex set {v1 , v2 } and edge set {e1 , e2 , e3 },
where the endpoints of e1 are v1 and v2 , the endpoints of e2 are v1 and v2 , and e3 is a loop at
v1 .
Solution:
We first draw the graph:

This graph has 11 subgraphs:

DEFINITION 6:
Let G be a graph and v a vertex of G. The degree of v, denoted deg(v), equals the number
of edges that are incident on v, with an edge that is a loop counted twice. The total degree
of G is the sum of the degrees of all the vertices of G.
EXAMPLE:
Find the degree of each vertex of the graph G shown below. Then find the total degree of G.

Solution:

THEOREM:
If G is any graph, then the sum of the degrees of all the vertices of G equals twice the number
of edges of G.
COROLLARY 1:
The total degree of a graph is even.
EXAMPLE:
Draw a graph with the specified properties or show that no such graph exists.
(a) Graph with four vertices of degrees 1, 1, 2, and 3.
(b) Graph with four vertices of degrees 1, 1, 3, and 3.
(c) Simple graph with four vertices of degrees 1, 1, 3, and 3.
Solution:

COROLLARY 2:
In any graph there are an even number of vertices of odd degree.
PROBLEM: Is it possible in a group of 9 people for each to shake hands with exactly 5 other
persons?
Solution: The answer is no. In fact, imagine a graph in which each of the 9 people is represented by a dot and two dots are joined by an edge if, and only if, the people they represent
shook hands. Suppose each of the people shook hands with exactly 5 others. Then we have an
odd number (nine) vertices of odd degree. This contradicts Corollary 2. 

Vous aimerez peut-être aussi